Download as pdf or txt
Download as pdf or txt
You are on page 1of 154

www.MRCPass.

com

Neurology


elzohryxp@yahoo.com https://www.facebook.com/elzohryxp

May2012

www.MRCPass.com

Neurology

NeurologyQ001

A 55 year old man patient presents with an 8 month history of progressive difficulty in swallowing, and dysarthria. He has lost 6 kg in weight. On examination he has a fasciculationonthetongueandabriskjawjerk. Whatisthelikelydiagnosis?
A.Amyotrophiclateralsclerosis B.Subacutecombineddegenerationofthecord C.Seniledementia D.Multiplesclerosis E.GuillainBarresyndrome

Answer:a)amyotrophiclateralsclerosis. A fifth of patients with amyotrophic lateral sclerosis have bulbar involvement. This is a classicpresentationofapatientwiththisvariantofmotorneurondisease.
Dr.KhalidYusufElzohrySohagTeachingHospital2012 2

www w.MRCPass.c com

Ne eurology

Ne eurologyQ002
A 60 0 year old man is brought has ha ad 6 seizur res over the e past week k. The seizu ures are generalisedton nicclonic. hascomplainedofase evereheada ache,andexamination e nshowsincr reasedtone eonthe Heh left with mild hemiparesis h s. There is p papillodoem ma in both e eyes. An em mergency MRI M scan emisphere. showsamassintherightcerebralhe atisthemo ostlikelydia agnosis? Wha
A.Medu ulloblastoma a B.Craniopharyngiom ma C.Meningioma blastoma D.Gliob E.Astroc cytoma

Answer:d)glio oblastoma. The e rapid onse et of sympto oms make it i likely that t this is a malignant m lesion, and th he most com mmon mali ignancy of f the cent tral nervou us system is a gliom ma. Gliobl lastoma mul ltiformeisby b farthemost m commo onandmos stmalignant toftheglia altumors.

Glio oblastoma

Dr.Khalid K Yusuf fElzohrySo ohagTeachin ngHospital 2012 3

www.MRCPass.com

Neurology

NeurologyQ003
A 42 year old man has weakness in the extensors of the upper limb and flexors of the lowerlimbmuscles.Hisspeechhasanasalquality.Thereistonguefasciculation. Whatisthelikelydiagnosis?
A.Wilson'sdisease B.CreutzfeldtJakobdisease C.Motorneurondisease D.Myastheniagravis E.Thyrotoxicosis

Answer:c)Motorneurondisease. In motor neuron disease, UMN signs include muscle spasticity, slowed recruitment of voluntary muscle strength, weakness especially in the extensors of the upper limb and flexorsofthelowerlimbmuscles,pseudobulbarpalsy. LMN features include muscle wasting and fasciculation, depressed reflexes and bulbar palsy. Onset of the disease is usually in mid to late adult life with the incidence increasingwithadvancingage.
Dr.KhalidYusufElzohrySohagTeachingHospital2012 4

www w.MRCPass.c com

Ne eurology

Ne eurologyQ004
Followingafoo otballinjury y,amanhasdeveloped dfootdrop pandhaslo ostsensationtothe dors salpartofthe t foot. Whi ichnerveismostlikely ytohavebeeninvolved d?
A.Tibial B.Sciatic C.Comm monperonea al D.Femo oral E.Gluteal

Answer:c)com mmonperon neal. The e common peroneal nerve n controls foot eversion e an nd dorsiflexion. Sensa ation is suppliedtothe eanterolateralpartof fthelegand dthedorsumofthefoot.

Dr.Khalid K Yusuf fElzohrySo ohagTeachin ngHospital 2012 5

www w.MRCPass.c com

Ne eurology

Ne eurologyQ005
A 70 year old woman wi ith a histor ry of hyper rtension and insulinde ependent diabetes d mellitus prese ented with new onset of uncon ntrolled vio olent movements of the t left extr remitiesacc companiedbyheadach hes. Neu urological examination e n was nor rmal except for the constant, relentless, violent mov vement of her left arm m and leg. Although she was full ly awake, alert, and oriented, shecouldnotstop s theabn normalmov vementsex xceptforashort s period doftime. spresentati ionislikelyto t bedueto oalesionin nthe: This
A.Subst tantianigra B.Globu uspallidus C.Putam men D.Hypothalamus E.Subth halamicnucle eus

Answer:e)subthalamicnu ucleus. Hem miballismus s is a rare d disorder cha aracterised by involun ntary wild fl linging mov vements ofthelimbs.Itisusuallyu unilateral(h hencehemiballismus)and a iscause edbylesion nsinthe contralateralsu ubthalamicnucleus.

Dr.Khalid K Yusuf fElzohrySo ohagTeachin ngHospital 2012 6

www.MRCPass.com

Neurology

NeurologyQ006
A 20 year old lady presents with a history of a single tonicclonic seizure. She had not been taking any illicit drugs. She also reports the occurrence of occasional absent spells andbriefjerkingofhisupperlimbs,whenshehasbeenoutlatepartying. WhichoftheFollowingdrugsismostappropriate?
A.Lorazepam B.Carbamazepine C.Phenytoin D.Gabapentin E.Sodiumvalproate

Answer:e)sodiumvalproate. This patient is likely to have juvenile myoclonic epilepsy (JME), and valproate is the treatment of choice. Juvenile myoclonic epilepsy (JME) is an idiopathic generalized epileptic syndrome characterized by myoclonic jerks, generalized tonicclonic seizures (GTCSs),andsometimesabsenceseizures. These can be precipitated when the patient is sleep deprived. Apart from sodium valproate,lamotrigineandtopirimatecanalsobeused.
Dr.KhalidYusufElzohrySohagTeachingHospital2012 7

www.MRCPass.com

Neurology

NeurologyQ007
A30year old patient has recurrentepisodesofsevere, unilateral, painfelt in theregion oftheeyeandforeheadassociatedwithlacrimation. Themostlikelydiagnosisis:
A.Migraine B.Subarachnoidhaemorrhage C.Retroorbitaltumour D.Clusterheadache E.Migranousneuralgia

Answer:d)clusterheadache. Clusterheadachecausessevere,usuallyunilateral,painfeltintheregionoftheeyeand forehead associated with lacrimation, conjunctival injection and occasionally transient Hornerssyndrome. Oxygen and triptans may arrest attacks, prophylaxis with propranolol, pizotifen, and especiallyverapamilmaypreventfurtherepisodes.
Dr.KhalidYusufElzohrySohagTeachingHospital2012 8

www w.MRCPass.c com

Ne eurology

Ne eurologyQ008
A 70 year old man has a 2 month h history of progressive p weakness in his legs. He has wea akness in th he hip and knee distribution. Kne ee and ank kle reflexes are brisk. Plantars P areupgoingbil laterally.He ehassensorylossfrom mlegsupwardstowardstheT5lev vel. Whi ichoneofth heFollowin ngisalikely ydiagnosis? ?
A.Subac cutecombinedgeneratio onofcord B.Friedr rich'sataxia C.Meningioma D.Hered ditaryspastic cparaparesis E.Tropic calspasticpa araparesis

Answer:c)men ningioma. A meningioma m involving h high in the spinal cord d (thoracic region) can n cause a T5 T level. The erestareallcausesof spasticpara aparesis butare unlike ely to cause e a sensorylevel on clinicalexamination.

Spin nalCordMe eningioma

Dr.Khalid K Yusuf fElzohrySo ohagTeachin ngHospital 2012 9

www.MRCPass.com

Neurology

NeurologyQ009
A 50 patient is assessed for gradually progressive dementia over several months. Whilst hewasonthewardhewasnoticedtohavemyoclonicjerksofhishands. Whatisthelikelycause?
A.HIVencephalopathy B.Parkinsonsdisease C.Alzheimersdisease D.Picksdisease E.CreutzfeldtJakobdisease

Answer:E)CreutzfeldtJakobdisease Creutzfeldt Jakob disease (CJD) usually occurs in the 4575 year age group and most commonlypresentsasarapidlyevolvingmultifocaldementiawithmyoclonicjerksinthe latter stages. New variant CJD, which has been strongly linked to infection from meat products of cows with BSE, tends to present with behavioural and psychiatric disturbancesprogressingtoincoordinationanddementiawithmyoclonicjerks.
Dr.KhalidYusufElzohrySohagTeachingHospital2012 10

www.MRCPass.com

Neurology

NeurologyQ010
A 45 year old man has presented with lower back pain, radiating to his buttocks, associated with lower limb parathesia over the last few days. He is unable to walk due tolegweakness.Theanklereflexesareabsent. Whatisthediagnosis?
A.Discprolapse B.Transversemyelitis C.GuillainBarresyndrome D.Multiplesclerosis E.Friedrich'sataxia

Answer:c)GuillainBarresyndrome. Guillain Barr syndrome is preceded by respiratory or gastrointestinal symptoms in twothirds, but not all, cases. Lower back pain, often radiating to the buttocks occurs in athirdofcases.Difficultywalkinginthiscaseisduetodistalweaknessandabsentdistal reflexessuggestGBS.
Dr.KhalidYusufElzohrySohagTeachingHospital2012 11

www.MRCPass.com

Neurology

NeurologyQ011
A62yearoldmanpresentswitharestingtremorofhisrightarm.Hewasfoundtohave cogwheelingandbradykinesia.Hisgaitisshufflinginnature. WhichoneoftheFollowingdrugsismostlikelytohelphertremor?
A.Amantadine B.Benzhexol C.Bromocriptine D.CoCareldopa E.Selegiline

Answer:B)Benzhexol. Benzhexol is an anticholinergic drug (used to alleviate tremors in parkinsons disease). The first line treatment is with Ldopa which is the metabolic precursor of Ldopa. Benzhexolisnoteffectiveagainstbradykinesia.
Dr.KhalidYusufElzohrySohagTeachingHospital2012 12

www.MRCPass.com

Neurology

NeurologyQ012
A 46 year old female had a traumatic left sided tooth extraction 6 years ago. Since then she is complaining of facial pain mainly over upper part of left face with intermittent exacerbation.Occasionallythepainradiatestorightsideoftheface. Whatisthelikelydiagnosis?
A.Atypicalfacialpain B.Trigeminalneuralgia C.Drysocketcyst D.Bellspalsy E.Periodontitis

Answer:A)atypicalfacialpain. Complicated dental procedures or other forms of trauma can lead to a form of atypical facial pains which is also known as post traumatic facial pain/ neuralgia. The pain is usuallyselflimitingafterseveralyears.
Dr.KhalidYusufElzohrySohagTeachingHospital2012 13

www.MRCPass.com

Neurology

NeurologyQ013
A 25 year old secretary has had several episodes of brief jerking of the right arm over thepastfewweeks. Thereisnolossofconsciousness.ACTscanoftheheadisunremarkable. Whichisthebestmedicationtocommence?
A.Carbamazepine B.Phenytoin C.Lorazepam D.Diazepam E.Levodopa

Answer:a)carbamazepine. Brief episodes of jerking suggests simple partial seizures. Carbamazepine is first line therapyforthis.
Dr.KhalidYusufElzohrySohagTeachingHospital2012 14

www.MRCPass.com

Neurology

NeurologyQ014
A 63 year old man who has been diagnosed with a glioma is commenced on chemotherapy.4dayslater,hebeginstobehavestrangely,andhassuicidalideation. WhichoneoftheFollowingismostlikely?
A.Vincristineencephalitis B.Hyponatraemia C.Steroidpsychosis D.Hypoglycaemia E.Hypocalcaemia

Answer:C)steroidpsychosis. Incidence of steroid associated cognitive changes including psychosis are high particularlywhenhighdosesteroidsareused(e.g.dexamethaxone).
Dr.KhalidYusufElzohrySohagTeachingHospital2012 15

www w.MRCPass.c com

Ne eurology

Ne eurologyQ015
A 75 7 year old d man was s admitted to the ho ospital after being un nable to co ope. His neig ghbours say y that he had h been in ncreasingly confused over o the last month and a two wee eks ago wa as seen to have a gen neralised se eizure. He h had no neu urological signs s on examinationap partfromanupgoingplantar p ontheleft. ichoftheFo ollowingismost m likely? ? Whi
A.Subar rachnoidhae emorrhage B.Meningitis C.Subdu uralhaemato oma D.Parkinson'ssyndr rome E.Menin ngioma

Answer:c)subduralhaem matoma. A h history of confusion along with a possible seizure/fall in the eld derlly shou uld alert tow wardssubdu uralhaematoma.

Sub bduralHaem matoma

Dr.Khalid K Yusuf fElzohrySo ohagTeachin ngHospital 2012 16

www.MRCPass.com

Neurology

NeurologyQ016
A 65 year old man has a history of hypertension. He presents with sudden onset dysarthria,vomitinghiccupandvertigo. On examination, he has a right sided Horners syndrome, rightsided cerebellar ataxia, loss of pain and temperature sensation on the right hand side of the face, and loss of painandtemperaturesensationintheleftupperandlowerlimbs. Whatisthelikelydiagnosis?
A.Cerebellarinfarct B.Inferiorparietalinfarct C.Superiortemporalinfarct D.Subthalamicnucleusinfarct E.Lateralmedullaryinfarct

Answer:E)lateralmedullaryinfarct. Inthelateralmedullarysyndrome(inferiorcerebellararteryinvolvement),9thand10th nerve involvelement leads to dysphagia and dysarthria. There is alsoipsilateral Horners syndrome and facial sensory loss, and contralateral pain/ temperature sensory loss to upperandlowerlimbs.
Dr.KhalidYusufElzohrySohagTeachingHospital2012 17

www.MRCPass.com

Neurology

NeurologyQ017
A 60 year old man presents with drowsiness and confusion. A friend says he has been confabulating about having his wallet stolen. On examination, he is unkempt, his pupil reflexes are normal and eye movements are normal. There were otherwise no focal neurological deficits. A CT scan of the brain showed mild cerebral atrophy. He has a MCVof105fl. Whichdiagnosisismostlikely?
A.Transientglobalamnesia B.Korsakoff'spsychosis C.Wernicke'sencephalopathy D.Alzheimer's E.Pick'sdisease

Answer:b)Korsakoff'spsychosis. ThehighMCVsuggestsheavyalcoholdrinking.Heisdelusional(havinghiswalletstolen) and also there are no signs of nystagmus or cerebellar signs to support Wernicke's encephalopathy. Short term memory loss, amnesia and personality changes are also seen in Korsakoff's psychosis.
Dr.KhalidYusufElzohrySohagTeachingHospital2012 18

www.MRCPass.com

Neurology

NeurologyQ018
A 17 year old boy with mild learning disability presents for assessment. He was apparentlywell 5years ago,whenhedevelopedjaundice whichpersistedfor4months. He was found to have difficulty speaking, walking up the stairs and he also had choreoathetoidmovementsofthearms. On examination, there was jaundice, but no clubbing, cyanosis or peripheral lymphadenopathy. A neurological examination showed weakness and wasting of muscles acting on the shoulder and hip joints and brisk deep reflexes. The plantar responses were flexor. There were no extrapyramidal signs. The patient had Kayser Fleisherringsconfirmedbyslitlampexamination. Whatisthelikelydiagnosis?
A.NewvariantCJD B.Alzheimersdisease C.Motorneurondisease D.Wilsonsdisease E.Picksdisease

Answer:D)Wilsonsdisease. Wilson disease is a rare autosomal recessive inherited disorder of copper metabolism. The condition is characterized by excessive deposition of copper in the liver, brain, and othertissues. Wilsons disease usually presents at ages less than 40. Hepatic dysfunction is the presenting feature in more than half of patients. Apart from hepatitis, neuropsychiatric presentation and movement disorders are common presentations. KayserFleischer ringsareobservedinupto90%ofindividualswithsymptomaticWilsondisease. Manifestations include dystonia, choreoathetoid movements, spasticity, grand mal seizures,rigidity,andflexioncontractures.
Dr.KhalidYusufElzohrySohagTeachingHospital2012 19

www.MRCPass.com

Neurology

NeurologyQ019
A 60 year old man is on high dose prednisolone for a diagnosis of giant cell arteritis 4 months ago. He complains of a headache, which had worsened for the past week and doublevision. Onexamination,thereisneckstiffness,andrightsided7thnervepalsy.Histemperature is 38 degrees. A lumbar puncture reveals a protein of 0.8 g/l, glucose of 3.5, WCC of 30 (predominantneutrophils). Whatisthemostlikelycausativeorganism?
A.Listeriamonocytogenes B.Mycobacteriumtuberculosis C.Mycoplasmapneumoniae D.Borreliaburgdoferi E.Herpessimplex

Answer:a)listeriamonocytogenes. The lumbar puncture findings are consistent with a bacterial meningitis but the glucose is not low enough to suggest TB. Listeria monocytogenes is an aerobic and facultatively anaerobicgrampositivebacillus.Theriskoflisteriosisismarkedlyincreasedinimmuno compromised patients, particularly among those undergoing renal transplantation, receivinghighdosesofcorticosteroids,orsufferingwithAIDSorcancer. Ampicillinorpenicillinhasgenerallybeenrecommendedasthetreatmentofchoice.
Dr.KhalidYusufElzohrySohagTeachingHospital2012 20

www.MRCPass.com

Neurology

NeurologyQ020
A 45 year old lady presents with a week's history of spontaneous, deep, right shoulder painradiatingtotheneck. There is also weakness and then wasting of the right deltoid, spinati and triceps muscles. Whichisthelikelydiagnosis?
A.Spinobulbardystrophy B.C5andC6myelopathy C.Guillainbarresyndrome D.Rotatorcufftendonitis E.Brachialneuritis

Answer:e)brachialneuritis. Idiopathic brachial neuritis is an immunemediated disorder, often preceded by an upper respiratory tract infection or immunisation. The initial feature is the abruptonset of unilateral arm pain or shoulder pain. There is also evolving weakness, which is worst 23weeksaftertheonsetofpain.
Dr.KhalidYusufElzohrySohagTeachingHospital2012 21

www.MRCPass.com

Neurology

NeurologyQ021
A 60 year old man presents with difficulty with his mobility. He has increased tone, bradykinesia,apillrollingtremorandashufflinggait. Whichmedicationshouldbecommencedfirst?
A.Benztropine B.Amantadine C.Selegiline D.Apomorphine E.Cocareldopa

Answer:e)cocareldopa. Cocareldopa is the first medication used to increase dopaminergic activity in the basal ganglia. Apomorphine is used for onoff fluctuations. Benztropine is used for anticholinergic side effects. Selegiline is an MAO inhibitor, and Amantadine is an antiviral drug. Both are used as a second line drugs. Apomorphine is used for onoff fluctuations.
Dr.KhalidYusufElzohrySohagTeachingHospital2012 22

www.MRCPass.com

Neurology

NeurologyQ022
A 50 year old man presents with a 5 year history of headaches. The pain has gradually worsenedandisnowpresentdaily,particularlyonwaking.Hedescribesthepainasdull, generalised and constant. It is exacerbated by bright light. Neurological examination is unremarkable.Sheneedstotakeatleasttwoparacetamolandtwotramadoltabletsper day. Whichisthelikelydiagnosis?
A.Clusterheadaches B.Analgesicinducedheadaches C.Migrainousheadaches D.Spaceoccupyingtumour E.Trigeminalneuralgia

Answer:b)analgesicinducedheadaches. Frequent use of some immediaterelief medications can result in recurring or persistent headache in those with preexisting headache and an individual susceptibility. Three or more simple analgesics (aspirin and/ or acetaminophen) a day (more than 1000 mg) moreoftenthan5daysaweek. Frequent use of shortacting NSAIDs such as ibuprofen and also opiate based drugs can alsobeacause.
Dr.KhalidYusufElzohrySohagTeachingHospital2012 23

www.MRCPass.com

Neurology

NeurologyQ023
A 60 year old patient has diplopia looking to the left. When the left eye is covered, the outerimagedisappears. Whichnerveisinvolved?
A.Leftabducens B.Rightabducens C.Lefttrochlear D.Righttrochlear E.Leftoculomotor

Answer:a)leftabducens. Covering the affected eye causes the false image (outer image) to disappear. In this case,thelefteyeisinvolvedhenceleftlateralrectus(abducensnerve).
Dr.KhalidYusufElzohrySohagTeachingHospital2012 24

www.MRCPass.com

Neurology

NeurologyQ024
A22yearoldmanhadanaccident3yearsago.Duringthattimehewasdocumentedto haveaT12lesiononthespinalcord.Henowpresentswithnumbnessonhistrunk. Onexamination,thereisreducedsensationtopinprickfromT6toT10. Whatisthemostlikelycauseofthis?
A.Brownsequardsyndrome B.Subacutecombineddegenerationofthecord C.Anteriorspinalarterythrombosis D.Posttraumaticsyrinx E.Arteriovenousmalformation

Answer:D)posttraumaticsyrinx. Posttraumaticsyringomyelia(PTS)referstothedevelopmentandprogressionofacyst filled with cerebrospinal fluid (CSF) within the spinal cord. Other symptoms include increased weakness, numbness, increased spasticity, and hyperhidrosis (increased sw eating). Ascendingsensorylevelandsensorydissociation(selectivelossofpainandtemperature sensation) are very sensitive indicators for detecting progressive PTS. MRI is the preferredinitialimagingstudyforthediagnosisofPTS.
Dr.KhalidYusufElzohrySohagTeachingHospital2012 25

www.MRCPass.com

Neurology

NeurologyQ025
A 42 year old woman presents with a history of headaches. She also reports menstrual irregularities. On examination, she was noted to be obese. Fundoscopy revealed bilateralpapilloedema.Tone,powerandreflexesinthelimbswerenormal. Investigations show a normal CT scan and MRI. EEG normal. Lumbar puncture reveals elevatedCSFpressurebuttheconstituentsofCSFarenormal. In this particular patient, which of the Following would suggest that a complication has arisen?
A.Bitemporalhemianopia B.Lossofcolourvision C.4thnervepalsy D.Visualloss E.Torticollis

Answer:d)visualloss. Thediagnosisisidiopathicintracranialhypertension(benignintracranialhypertesion). Inthisconditionpapilloedemamayresultinanenlargedblindspot. Uniocular visual loss would suggest infarction of the optic nerve consequent on severe andlongstandingpapilloedema.
Dr.KhalidYusufElzohrySohagTeachingHospital2012 26

www.MRCPass.com

Neurology

NeurologyQ026
A 70 year old man presents with sudden onset dysphagia and dysarthria, vomiting hiccupandvertigo. On examination he has a right sided Horners syndrome, rightsided cerebellar ataxia, loss of pain and temperature sensation on the right hand side of the face and loss of painandtemperaturesensationintheleftupperandlowerlimbs. Whereisthelesion?
A.Pons B.Lateralmedulla C.Cerebellum D.Midbrain E.Tectum

Answer:b)lateralmedulla. Thepatienthasthelateralmedullarysyndrome. Multiple areas are involved : 9th and 10th nerve dysphagia and dysarthria Vestibular nuclei vertigo Inferior cerebellar peduncle ipsilateral cerebellar ataxia Descending autonomicfibresHornerssyndromeFifthnervenucleuslossofpainandtemperature sensation over the face (ipsilateral) Lateral leminiscus loss of pain and temperature sensationinthecontralaterallimbs
Dr.KhalidYusufElzohrySohagTeachingHospital2012 27

www.MRCPass.com

Neurology

NeurologyQ027
Apatientpresentswithweaknessofkneeextensionandankleinversion. WhichoftheFollowingnerverootscouldbedamaged?
A.L2 B.L3 C.L4 D.L5 E.S1

Answer:c)L4. L4 is involved in knee extension and ankle inversion. The tibial nerve carries L4 & L5 roots.
Dr.KhalidYusufElzohrySohagTeachingHospital2012 28

www w.MRCPass.c com

Ne eurology

Ne eurologyQ028
A 25 5 year old man m has come fromM Mexico 5 yea ars ago.Sinc ce a year ag go, he has had h two toni icclonicseizuresawee ek. On examinatio on, he appe ears well, w with no focal neurolog gical deficit. A CT scan n shows mul ltiplecalcifiedcysticles sionsinthe ebrain. ichdiagnos sisislikely? Whi
A.Neuro osarcoid B.Neuro ocysticercosis C.Multiplesclerosis D.Cereb braltoxoplas smosis E.Tuber rculosis

Answer:b)neu urocysticerc cosis. Neu urocysticerc cosisiscaus sedbyTaen nia solium(pork tapew worm). Theremay beseizures s due e to localise ed inflamm mation that t accompan nies their degeneratio d on in the cerebral c cort texwhenca alcifiedcystsoccur.Thi isdiseaseis sfoundinSo outhAmeri icaandAsia a. Neu urocysticerc cosistypical llyisbenign n,andmostlesionsreso olvesponta aneouslywithin23 mon nths. An en nzymelinked d immunotransfer blo ot (EITB) ass say of a pa atient's seru um may confirmthedia agnosis.Alb bendazoleis stherecom mmendedtre eatment.

Neu urocysticerc cosis

Dr.Khalid K Yusuf fElzohrySo ohagTeachin ngHospital 2012 29

www.MRCPass.com

Neurology

NeurologyQ029
A 42 year old lady presents with weakness, diplopia and fatigue. She has a past medical historyofrheumatoidarthritis. On examination there was bilateral partial ptosis and weakness of abduction of both eyes. Whatisthelikelydiagnosis?
A.Myastheniagravis B.Mononeuritismultiplex C.GuillainBarrsyndrome D.Paraneoplasticsyndrome E.LambertEatonmyasthenicsyndrome

Answer:a)Myastheniagravis. The most likely diagnosis is myasthenia gravis. There is an association between myasthenia gravis, pernicious anaemia, systemic lupus erythematosis and rheumatoid arthritis. The condition is more common in women with a peak incidence around the age of 30. It characterised by fatiguability of the proximal limb muscles, ocular and bulbarmuscles.Reflexesareinitiallypreservedbutmaybefatiguable.
Dr.KhalidYusufElzohrySohagTeachingHospital2012 30

www.MRCPass.com

Neurology

NeurologyQ030
A 30 year old man presents with a 6 month history of recurrent episodes altered beaviour. During these episodes, he develops a motionless stare with associated lip smacking,grimacing,chewingmovements,scratchingorgesturing.Hispartnerdescribes himashavinglabileemotions,hemaygetsuddenoutburstsofaggressionoragitation. WhichoftheFollowingisthelikelydiagnosis?
A.Transientglobalamnesia B.Frontallobeepilepsy C.Temporallobeepilepsy D.Parietallobelesion E.Migraine

Answer:c)temporallobeepilepsy. The features of seizures beginning in the temporal lobe can be extremely varied, but certain patterns are common. In temporal lobe epilepsy, there may be a mixture of different feelings, emotions, thoughts, and experiences, which may be familiar or completely foreign. Temporal lobe epilepsy may, for example, cause sudden outbursts of unexpected aggression or agitation, or it may be characterized by auralike phenomena. Complexpartialseizuresarecharacterizedbyimpairedawareness.Theyloseawareness and tend to have a motionless stare accompanied by automatisms stereotyped, repetitive, involuntary movements such as lip smacking, chew ing, picking at objects, scratching,andgesturing. In some cases, a series of old memories resurfaces. Hallucinations of voices, music, people, smells, or tastes may occur. These features are called auras or warnings. Theymaylastforjustafewseconds,ormaycontinueaslongasaminuteortwo. Carbamazepineandphenytoinareusedtotreatthecondition.
Dr.KhalidYusufElzohrySohagTeachingHospital2012 31

www.MRCPass.com

Neurology

NeurologyQ031
A 55 year old drug user who is homeless presents with lethargy. General examination revealspoorhygieneandasacralsore.Hehasbilateralptosis,anddifficultyswallowing. Thereisalsogeneralweaknessinalllimbs.Hislethargyisworseintheevenings. WhichoftheFollowingisthelikelydiagnosis?
A.Myastheniagravis B.Lamberteatonmyasthenicsyndrome C.Botulism D.Motorneurondisease E.HIVneuropathy

Answer:c)botulism. The main differentials are myasthenia gravis, LEMS and botulism, but in this context botulism is more likely. Botulism is a paralytic disease caused by the neurotoxins of Clostridium botulinum. Wound botulism, caused by systemic spread of toxin produced byorganismsinhabitingwounds,trauma,surgeryandsubcutaneousheroininjection. The neurologic symptomatology often has been described as a progressive, descending weakness or paralysis that affects muscles innervated by the cranial nerves. Respiratory difficulty arises from airway obstruction and diaphragmatic weakness. Diplopia, dysarthria, dry mouth, and generalized weakness are among the most common presentingsymptoms.
Dr.KhalidYusufElzohrySohagTeachingHospital2012 32

www.MRCPass.com

Neurology

NeurologyQ032
A 50 year old teacher develops a sided facial weakness in association with hearing loss andpainintherightear. On examination, there is a vesicular rash over the right ear, and right lower motor neuron7thnervepalsy. Whatisthelikelycause?
A.Lymedisease B.Herpeszoster C.Diabetes D.Polyarteritisnodosa E.Syphilis

Answer:b)herpeszoster. ThisisRamsayHuntsyndrome.TheprimarypathophysiologyofRamsayHuntsyndrome islocatedinthegeniculateganglionoftheseventhcranialnerve(CNVII). Classically, Ramsay Hunt syndrome has been associated with VZV. It is associated with 7thnervepalsy,vertigo,ipsilateralhearinglossandtinnitus.
Dr.KhalidYusufElzohrySohagTeachingHospital2012 33

www.MRCPass.com

Neurology

NeurologyQ033
A 55 year old man has left sided hearing loss and vertigo. On examination, he has an absentcornealreflexontheleft.Thereisalsomildleftsidedfacialweakness. WhichoneoftheFollowingismostlikely?
A.Frontallobetumour B.Cavernoussinusthrombosis C.RamsayHuntsyndrome D.Cerebellopontineangletumour E.Syringomyelia

Answer:D)cerebellopontineangletumour. Cerebellopontine angle (CPA) tumors can cause vertigo, unilateral hearing loss. Large tumors may cause subtle facial weakness, decreased corneal reflex, and facial dysesthesia. If there was opthalmoplegia or chemosis, then a cavernous sinus thrombosiswouldbemorelikely(itcanalsocauseabsentcornealreflexes).
Dr.KhalidYusufElzohrySohagTeachingHospital2012 34

www w.MRCPass.c com

Ne eurology

Ne eurologyQ034
A 60 6 year old d woman with w headac che and na ausea is su uspected of having posterior cere ebralartery ythrombosis. Whi ichoftheFo ollowingisarecognise edfeatureofthisoccur rrence?
A.Cereb bellarataxia B.Hemiparesis C.Homo onymoushem mianopia D.Thirdnervepalsy E.Sixthnervepalsy

Answer:c)hom monymoushemianopia h a. The eposteriorcerebral c art terysupplie estheoccip pitallobean ndocclusion ncausesdam mageto thevisualcorte ex,resulting ginhomony ymoushem mianopia.

Post teriorCereb bralArteryInfarct

Dr.Khalid K Yusuf fElzohrySo ohagTeachin ngHospital 2012 35

www.MRCPass.com

Neurology

NeurologyQ035
A35yearold womanwakes up with a sudden onset severe sharp headache.Shehas no neurologicalsigns.CTofherheadisnormal. Whatisthebestnextinvestigation?
A.MRV B.Lumbarpuncture C.Serumelectrophoresis D.EEG E.Bonescan

Answer:b)lumbarpuncture. Thediagnosisofsubarachnoidhaemorrhageneedstobeexcludeinapatientwithacute sudden onset severe headache. The CSF sample should be sent for xanthochromia. Lumbar puncture is recommended 12 hours after the event to allow xanthochromia to develop. It is a yellow ish pigment (subtle and needs spectrophotometry) which indicatesthepresenceofbilirubinintheCSF.
Dr.KhalidYusufElzohrySohagTeachingHospital2012 36

www.MRCPass.com

Neurology

NeurologyQ036
A30yearoldmanhasahistoryofepilepsy.Hewasfoundonthestreethavingaseizure and was brought to A+E. Rectal diazepam had been given by the ambulance crew. His seizurelastsmorethan15minutes. Whatisthebestmanagementstep?
A.Intravenouslorazepam B.Intravenousphenytoin C.Oralcarbamazepine D.Intravenousphenobarbitone E.CTscanofthehead

Answer:b)intravenousphenytoin. Thispatienthasstatusepilepticus.Hehasnotrespondedtoabenzodiazepineandhence thenextstepistoloadwithintravenousphenytoinatadoseof15mg/kg.


Dr.KhalidYusufElzohrySohagTeachingHospital2012 37

www w.MRCPass.c com

Ne eurology

Ne eurologyQ037
A60 0yearoldman m isaveg getarianand dpresentswith w letharg gy.Hehasfrequent f diarrhoea andmentionsthat t heeats smostlyma aize. on, he has an erythema atous rash across a his face and che est. He is co onfused On examinatio and disorientat ted. On exa amination, he h has an MMSE M score e of 21 /30. . Tone and reflexes arenormalbut theweakth hroughoutthe t body. Whi ichvitaminishisdietli ikelytobed deficient?
A.Thiam mine B.B12 C.Niacin n D.Vitam minC E.Vitam minA

Answer:c)niac cin. The e diagnosis is i pellagra. There is a triad of dem mentia, dia arrhoea and d dermatitis s. Niacin (nicotinamide or nicotinic c acid) defic ciency causes pellagra only if tryp ptophan, an n amino acid d, is also de eficient. Peo ople who li ive in areas s where ma aize (Indian n corn) is th he main food d source are a at risk of develop ping pellagr ra because e maize is low in niacin and tryp ptophan. Pellagra affect ts the skin, digestive t tract, and brain. b A ph hotosensitiv ve rash may y occur. Skin nabnormali itiesareper rsistent,and dtheaffect tedareasmay m become ebrownand dscaly. The e whole dig gestive tra act is affec cted. Other r symptom ms include nausea, vo omiting, constipation, and diar rrhea. Late er, fatigue e, insomn nia, and apathy develop. d Encephalopath hy usually follows. It is chara acterized by b confusio on, disorientation, hallucinations,andmemo oryloss.

Pho otosensitiverashseeni inPellagra


Dr.Khalid K Yusuf fElzohrySo ohagTeachin ngHospital 2012 38

www.MRCPass.com

Neurology

NeurologyQ038
A 45 year old woman has sensory loss and wasting of the small hand muscles. One examination, she also has a right sided Horner's syndrome. Her arms demonstrate thickeningof the subcutaneous tissues. There isalso evidence of Charcots jointson the wrists. Whatisthediagnosis?
A.Hereditaryspinocerebellarataxia B.Hereditaryspasticparaparesis C.Motorneurondisease D.Syringomyelia E.Multiplesclerosis

Answer:d)Syringomyelia. Syringomyelia is chronic disorder characterised by the presence of gliallined cavities situatedinthecentralpartofthespinalcord. Recognised causes include Chiari type I malformation, central cord tumours, basal arachnoiditisandtrauma. There may be sensory loss, wasting of the small hand muscles, uni or bilateral Horners syndrome, abnormalities of sw eating, thickening of subcutaneous tissues, atrophy and decalcificationofbones,developmentofCharcotsjointsandChiariImalformation(due toarachnoiditis).
Dr.KhalidYusufElzohrySohagTeachingHospital2012 39

www.MRCPass.com

Neurology

NeurologyQ039
A27year old lady presentswith a severe headache, which woke her up fromsleep. She is not pregnant and was on no drugs in particular she was not on hormonal contraception. On examination she was afebrile and alert. On examination of the optic fundus the cup of the optic disc was filled and the medial margins of the disc were blurred. There was no other CNS abnormality, in particular no neck stiffness. A CT scan wasasnormal. Whatisthelikelydiagnosis?
A.Meningioma B.Sagittalsinusthrombosis C.Benignintracranialhypertension D.Migraine E.Meningitis

Answer:b)sagittalsinusthrombosis. This is the most likely cause, despite not being pregnant or on the OCP. In only 10% of cases is cerebral venous thrombosis due to damage to the vessel wall by infection, tumourortrauma. Commonest causes are inherited disorders of coagulation of which factor V Leiden mutation is found in around 20% of cases. Often there is combination of causative factors: e.g. protein S deficiency and child birth, pregnancy and Behcet's disease, OCP andfactorVLeidenmutation.
Dr.KhalidYusufElzohrySohagTeachingHospital2012 40

www.MRCPass.com

Neurology

NeurologyQ040
A 25 year old female patient presents with a 5day history of ascending muscle weaknessinbothhandsandfeet.EMGsconfirmacutedemyelinatingsensoryandmotor neuropathy. Whattreatmentshouldbestarted?
A.Phenytoin B.Diazepam C.Amitriptyline D.Intravenousimmunoglobulin E.Pyridostigmine

Answer:d)intravenousimmunoglobulin. The diagnosis is Guillain Barre syndrome. It is preceded by diarrhea (e.g. campylobacter) and chest infections (e.g. mycoplasma) in two thirds of cases. Autonomic dysfunction and hyporeflexia are associated. Studies have shown that plasma exchange and IVIg are equally effective in treating people within two to four weeksofonsetofGBS.
Dr.KhalidYusufElzohrySohagTeachingHospital2012 41

www w.MRCPass.c com

Ne eurology

Ne eurologyQ041
A 32 3 year old d man has difficulty with w his vision. On examination e n he has im mpaired adductionofth herighteye elookinglef ft.Thelefteye e hasjerk kynystagmu us. Whi ichinvestigationismos stlikelytoy yieldadiagnosis?
A.Nerve econduction nstudies B.CTofthehead C.Paired dCSFandse erumforoligo oclonalband ds D.Serum mcopperandcaerulopla asmin E.Visualevokedpot tentials

Answer:c)pair redCSFand dserumforoligoclonalbands. The eclinicalsce enarioisinte ernuclearo ophthalmop plegia. This s is most co ommonly se een in multi iple sclerosis. MRI of the brain an nd CSF are the t best diag gnostictests.Inthissce enariothelesionisinthe t rightme ediallongitu udinalfascic culus.

Inte ernuclearop pthalmoplegiapatientlookingto otheleft


Dr.Khalid K Yusuf fElzohrySo ohagTeachin ngHospital 2012 42

www.MRCPass.com

Neurology

NeurologyQ042
A patient has, on examination, weakness in plantar flexion and foot inversion on the left.Healsoisunabletotiptoeonthesamefoot.Anklejerkisabsent. Whichnervelesionismostlikely?
A.Commonperonealnerve B.L4nerveroot C.Tibialnerve D.Sciaticnerve E.Femoralnerve

Answer:c)tibialnerve. Thetibialnervesuppliesthegastrocnemiusmuscleandleadstotheabovefindings.The commonperonealnervecausesweaknessofeversionanddorsiflexion.


Dr.KhalidYusufElzohrySohagTeachingHospital2012 43

www.MRCPass.com

Neurology

NeurologyQ043
A 50 year old man has subjectively diminished light touch and pinprick sensation in the left hand extending to above the elbow. Joint position sense is intact. He has difficulty distinguishing a cigarette from a pen using the right hand with his eyes closed. His two pointdiscriminationis11mm. Whereisthelesion?
A.Anteriorfrontal B.Posteriorfrontal C.Anteriorparietal D.Posteriorparietal E.Cingulategyrus

Answer:c)anteriorparietal. Thesensorysignsdescribedhereareindicativeofalesionoftheanteriorparietalcortex (mid postcentral gyrus). The cingulate gyrus is part of the limbic system (mood and emotions).
Dr.KhalidYusufElzohrySohagTeachingHospital2012 44

www.MRCPass.com

Neurology

NeurologyQ044
A 55 year old man has been progressively getting more confused. His symptoms began aboutthreeyearsagowhenhenoticedlegstiffness.Atpresenthisentirebodyfeelsstiff andhehasarestingtremorinthelimbs.Hiswritinghasbecomesmall.Overthelastsix months, he has developed hallucinations and reports seeing ghost figures, and is also becomingforgetful,ononeoccasionleavingthegascookerfireon. On examination, his minimental score was 10/30. He has increased tone throughout which is spastic and cogw heeling. There is a tremor in all limbs. Cranial nerve examination reveals mild restriction of conjugate upgaze eye movement. His gait was shufflingwithatendencytofallbackwards. Whatisthemostlikelydiagnosis?
A.Parkinson'sdisease B.Pick'sdisease C.Lewybodydementia D.Huntington'sdisease E.Motorneurondisease

Answer:c)Lewybodydementia. The combination of progressive cognitive decline, fluctuating symptoms, visual hallucinations, extrapyramidal signs (rigidity and bradykinesia more prominent than tremor) suggest Lewy body dementia. It is progressive. Patients are at risk of falls and syncope. Symptoms and signs of lew y body dementia probably result in part from disruptionofinformationflowfromthestriatumtotheneocortex,especiallythefrontal lobe. The cause is multifactorial. Altered neuromodulator and/or neurotransmitter levels (eg, acetylcholine,dopamine)influencethefunctionofmanyneuronalcircuits.
Dr.KhalidYusufElzohrySohagTeachingHospital2012 45

www.MRCPass.com

Neurology

NeurologyQ045
A 35 year old patient presented to the hospital with generalised unsteadiness and limb weakness. There was a history of ascending weakness, beginning five days before admission. On examination, she had distal weakness with decreased reflexes in the lowerlimbs. She had a lumbar puncture. The results of the CSF showed 7 x 10^9/l lymphocytes, no erythrocytes, and 1.2 g/l protein. EMGs showed reduction of mean conduction velocity andprolongeddistallatency(DL)wereobservedinthemedian,ulnar,andtibialnerves. Whatisthediagnosis?
A.Multiplesclerosis B.Viralmeningitis C.Syphilis D.Trigeminalneuralgia E.GuillainBarresyndrome

Answer:e)GuillainBarresyndrome. Conditions such as viral meningitis and multiple sclerosis cause mild protein elevation (above0.5g).However,GuillainBarresyndromecausesmarkedproteinelevation,often approachingormorethan1g.M In Guillain Barre syndrome, there is acute demyelination of the nerves, leading to reducedconductionvelocitiesontheEMGs.
Dr.KhalidYusufElzohrySohagTeachingHospital2012 46

www.MRCPass.com

Neurology

NeurologyQ046
A43yearoldmanpresentswithfrequentheadachesandlossoflibido.Hewasfoundto have hypopituitarism on investigation. The CT scan shows a pituitary tumour with suprasellarextension. WhichoftheFollowingstructuresislikelybecompressed?
A.Abducensnerve B.Hypothalamus C.Trochlearnerve D.Opticchiasm E.3rdVentricle

Answer:D)Opticchiasm Superior extension (suprasellar) of a pituitary tumour can lead to compression and invasionoftheopticchiasmandnerve.
Dr.KhalidYusufElzohrySohagTeachingHospital2012 47

www.MRCPass.com

Neurology

NeurologyQ047
A32yearoldwomanhasknownmigraine.Shegetsperiodicepisodesofheadacheswith associatedvisualsymptoms. WhichoneoftheFollowingdrugsshouldbeusedfirstinamigraineattack?
A.Ibuprofen B.Methysergide C.Subcutaneoussumatriptan D.Oralsumatriptan E.Morphine

Answer:a)ibuprofen In acute migraine attack, the first line treatments are simple analgesics such as aspirin, ibuprofenorparacetamol. Second line treatment in acute migraine are the triptans (e.g sumatriptan) which work byselectivelystimulating5hydroxytriptamine1(5HT1)receptors.
Dr.KhalidYusufElzohrySohagTeachingHospital2012 48

www.MRCPass.com

Neurology

NeurologyQ048
A 30yearold woman has an 18 month history of unsteady gait, difficulty to speak and to perform fine movements with the fingers. She has a history of moderate alcohol intake. Neurological examination showed scanning dysarthria, horizontal nystagmus in the lateral gaze, severe bilateral dysmetria in the upper and lower limbs, bilateral dysdiadochokinesis,severegaitataxiaandinabilitytomaintainthesitting. Whatisthediagnosis?
A.Parkinson'sdisease B.Subacutecombineddegeneration C.Cerebellarsyndrome D.Motorneurondisease E.Myastheniagravis

Answer:c)cerebellarsyndrome. A mnemonic for cerebellar signs is VANISHD Vertigo, Ataxia, Nystagmus, Intention tremor,Scanningspeech,HypotoniaandDysdiadochokinesis. Cerebellarsyndromesarecommonlyduetoalcohol,cerebellarspaceoccupyinglesions, multiple sclerosis, and rarely, inherited sydromes such as Friedrich's Ataxia or Spinocerebellarataxia.
Dr.KhalidYusufElzohrySohagTeachingHospital2012 49

www.MRCPass.com

Neurology

NeurologyQ049
A 30 year old man presents with a 5 day history of weakness in the arms and legs, accompaniedbytingling.Hehadseveralepisodesofbloodydiarrhoeaillnesstwoweeks ago. Onexaminationtherewaslegandarmweaknessandflacciddeeptendonreflexes. Whichtestwouldbesthelpconfirmthediagnosis?
A.EMG B.EEG C.MRI D.HIVserology E.AntiAchantibody

Answer:a)EMG. The diagnosis is likely to be Guillain Barre syndrome. The two best tests are EMG (shows acute demyelination changes) and CSF (raised protein). Campylobacter serology shouldalsobesentforthispatient.
Dr.KhalidYusufElzohrySohagTeachingHospital2012 50

www.MRCPass.com

Neurology

NeurologyQ050
A 70 year old man who had multiple episodes of unilateral amaurosis fugax. He has risk factorsofbeingasmokerandhypertension.Heisonatenololandaspirin. AnECGshowsatrialfibrillation.Carotiddopplersshow80%rightcarotidstenosis. Whatisthemostappropriatemanagement?
A.Warfarin B.Carotidendarterectomy,thenwarfarinise C.Clopidogrel D.Highdoseaspirin300mgwithaprotonpumpinhibitor E.Tighthypertensivecontrol

Answer:b)carotidendarterectomy,thenwarfarinise. As there is > 70% carotid artery stenosis and symptoms suggestive of emboli, endarterectomyisrecommended.The patientshouldalsobe anticoagulated afterwards inviewofatrialfibrillation.
Dr.KhalidYusufElzohrySohagTeachingHospital2012 51

www w.MRCPass.c com

Ne eurology

Ne eurologyQ051
A50 0 yearoldman m has ah history ofhy ypertension nand is a sm moker. Hecomplains c of o visual loss s.Assessmentshowsth hepresence eofarighthomonymo h oushemiano opia. Whi ichstructur reisdamage ed?
A.Optic cchiasm B.Opticradiation C.Leftoccipital o lobe e D.Rightoccipitallob be E.Temp porallobe

Answer:c)leftoccipitallobe. Inhomonymou ushemianopia,thecon ntralateraloccipital o lob beisaffecte ed(usuallyinfarct).

Left toccipitalin nfarctonMRI

Dr.Khalid K Yusuf fElzohrySo ohagTeachin ngHospital 2012 52

www.MRCPass.com

Neurology

NeurologyQ052
A 66 year old man has had longstanding tremors in both his hands and forearms. Examination reveals normal tone, power and reflexes in his arms. The tremors improve whenhedrinksalcohol. Whatisthediagnosis?
A.Parkinson'sdisease B.Motorneurondisease C.Benignessentialtremor D.Prioninfection E.Hemiballismus

Answer:c)Benignessentialtremor. TheFollowingfeaturessupportadiagnosisofEssentialTremor: (1)bilateralactiontremorofthehandsandforearms (2)absenceofotherneurologicalsigns,exceptthecogwheelphenomenon (3)mayhaveisolatedheadtremorwithnosignsofdystonia Secondary criteria include a long disease duration (more than three years), a positive familyhistoryandbeneficialresponsetoalcohol(notanticholinergics).
Dr.KhalidYusufElzohrySohagTeachingHospital2012 53

www w.MRCPass.c com

Ne eurology

Ne eurologyQ053
A45 5yearoldman m hasdiff ficultygettingoutofth hechair. Onexaminatio e n,hehaspr roximalmuscleweakness. Inve estigationsshow s : Hb1 12.5g/dl MCV V79fl WCC7x10^9/ /l telets220x10^9/l plat urea a6mmol/l crea atinine110mol/l Crea atineKinase e7,000(24170)U/l atinvestiga ationshould dbedonene ext? Wha
A.Lumb barpuncture B.CTbrain C.MRIbrain b D.Musc clebiopsy E.Tensil lontest

Answer:D)musclebiopsy The eclinicalfeaturesareco onsistentw withdermato omyositisor o polymyos sitis. A su uitable area should be b identified d by electromyograph hy for musc cle biopsy. Muscle biop psy shows muscle ne ecrosis, pha agocytosis of o muscle fibres, and d an inflam mmatory infiltrate.

Poly ymyositisInflammato I oryinfiltrate esinamusc clebiopsy

Dr.Khalid K Yusuf fElzohrySo ohagTeachin ngHospital 2012 54

www w.MRCPass.c com

Ne eurology

Ne eurologyQ054
A 50 5 year old d lady com mplains of gradual on nset of blu urred vision n in her le eft eye. Examination re eveals a lef ft sided rela ative affere ent pupillar ry defect. Fundoscopy reveals c atrophy. Visual field ds show a le eft sided ce entral scoto oma and an n upper left sided optic quadranticvisu ualfielddef fectintherighteye. Whe ereisthele esion?
A.Leftoptic o nerve B.Leftanterior a optic cchiasm C.Sphen noidwing D.Leftoccipital o area a E.Leftoptic o radiatio on

Answer:b)leftanteriorop pticchiasm. . Lesi ions of the e anterior c chiasm (jun nction of op ptic nerve and chiasm m) will prod duce an ipsil lateral cen ntral scoto oma and a contralateral sup perior quadrantanopi ia. The contralateraldefectisdue etointerrup ptionofthe ecrossingnasalfibres.

Opt ticTract

Dr.Khalid K Yusuf fElzohrySo ohagTeachin ngHospital 2012 55

www.MRCPass.com

Neurology

NeurologyQ055
A 40 year old patient has been having fevers, neck stiffness and confusion. There was a pastmedicalhistoryofHIVinfectionanddiabetes. A CT scan was normal and lumbar puncture was performed. The opening pressure during LP was normal, and there were elevated CSF lymphocytes (88), and elevated CSF protein (5.5 g/l), and a low glucose. Gram staining and India ink preparation revealed 4 7m,roundbuddingyeastswithcapsuleand810lymphocytesperhighpowerfield. Whatisthediagnosis?
A.Mumpsmeningitis B.Mycobacteriumtuberculosis C.Cryptococcalmeningitis D.Carcinomatousmeningitis E.Meningococcalmeningitis

Answer:c)cryptococcalmeningitis. Cryptococcal meningitis is often seen in immunosuppressed patients. The organism is Cryptococcus Neoformans. Meningitis manifests with diffuse, nonfocal findings (eg, alteredmentalstatus,vomiting). A CT scan or MRI in patients with cryptococcal infection may reveal diffuse atrophy or cerebraledemawithfocal,homogenous,orcontrastenhancedareas. An India ink preparation is commonly used with CSF to identify the organism and to support a presumptive diagnosis. If performed correctly, 2550% of patients with cryptococcalmeningitisshowcryptococci. In patients with AIDS, amphotericin B is given for 2 weeks, with or without 2 weeks of flucytosine,followedbyfluconazoleat400mg/dforaminimumof10weeks.
Dr.KhalidYusufElzohrySohagTeachingHospital2012 56

www.MRCPass.com

Neurology

NeurologyQ056
A young lady visits her neurologist complaining of episodes of generalised weakness after argumentswith her partner.She alsocomplainsofseeinggoblinsuponwaking up. Atworkasasecretary,shehasdifficultystayingawakeandmayhavesleepattacks. Whatisthelikelydiagnosis?
A.Narcolepsy B.Generalisedepilepsy C.Petitmalseizures D.Obstructivesleepapnoea E.Jacksonianseizures

Answer:a)narcolepsy. Theconditiondescribedisnarcolepsy.Theepisodesdescribedarelikelytobecataplexy. TheHLAassociationisDQB1,Clomipramineisatricyclicantidepressantwhichmayhelp, thereisearlyREMsleepandhypnagogichallucinationsoccur.


Dr.KhalidYusufElzohrySohagTeachingHospital2012 57

www.MRCPass.com

Neurology

NeurologyQ057
A68yearoldwomanhasneckpainsandoccipitalheadachesfor2years.Sheisreferred to the neurology outpatients for assessment. Investigations showed a normal CT of the brain. Cervical X ray showed degenerative changes of narrow ed disc spaces and loss of cervicallordosis. Whatisthelikelydiagnosis?
A.Cerebellarhaemorrhage B.Temporalarteritis C.Occipitalneuralgia D.Epiduralhaemorrhage E.Cervicalspondylosis

Answer:E)cervicalspondylosis. Incervicalspondylosis,severaloverlappingsyndromesareseen:neckandshoulderpain, suboccipital pain and headache, radicular symptoms, and cervical spondylotic myelopathy. Examination findings include neck pain, radicular signs, and myelopathic signs. Cervical spine films can demonstrate disk space narrow ing, osteophytosis, loss of cervical lordosis, uncovertebral joint hypertrophy, apophyseal joint osteoarthritis, and vertebral canaldiameter.
Dr.KhalidYusufElzohrySohagTeachingHospital2012 58

www.MRCPass.com

Neurology

NeurologyQ58
A 45 year old woman is referred to you for investigation of headache. She has had headachesfor10years. Initially it responded to proprietary painkillers, but she is currently using the maximum dose of paracetamol, tramadol and diclofenac. The headaches are frequent throughout thedayandlastforhours.Therearenoassociatedvisualsymptoms. Whichisthenextbestmanagementstep?
A.Ivaspirin B.Caffeine C.Withdrawalofanalgesics D.Pizotifen E.Sumatriptan

Answer:c)withdrawalofanalgesics. The history of chronic use of analgesics and nature of headaches suggests analgesic inducedheadache.Insomepatientstheheadacheswillimprove.
Dr.KhalidYusufElzohrySohagTeachingHospital2012 59

www w.MRCPass.c com

Ne eurology

Ne eurologyQ59
A patient is un ndergoing examination e n of the ey ye. The patient has a direct response to light shone in the right eye, e but no o consensua al response e. Light sho one in the left eye elici itsaconsen nsualrespon nse,butnot tadirectre esponse. Dur ringpursuiteyemovem ments,thele efteyeisfix xedinaninferiorandlateralposit tion. ereisthele esion? Whe
A.Righttrochlearne erve B.Leftoptic o nerve C.Leftoculomotor o n nerve D.Rightabducentsnerve n E.Lefttr rochlearnerve

Answer:c)leftoculomoto ornerve. A le eft third ner rve palsy will w cause a dilated left t pupil, with h the eye in n a 'down and a out' position.Theafferentpathwayiscon ntrolledbythe t opticne erveandthe eefferentpathway p byt theoculomo otornerve henceadi ilatedpoorl lyreactingpupil. p

Left tThirdNerv vePalsy

Dr.Khalid K Yusuf fElzohrySo ohagTeachin ngHospital 2012 60

www.MRCPass.com

Neurology

NeurologyQ60
A 18 year old male is wheelchair bound and has difficulty with respiration. He also has upperlimbweakness. When he was younger he developed marked hypertrophy of his muscles. Blood tests revealaraisedcreatinekinase. Whatisamusclebiopsylikelytoshow?
A.Necroticmusclefibres B.Absenceofdystrophin C.Excessivelipidstorage D.Macrophageinfiltration E.Vasculiticchanges

Answer:b)absenceofdystrophin. Mutation in the dystrophin gene causes deficiency of dystrophin in Duchenne's muscular dystrophy. Patients develop progressive upper and lower limb weakness with pseudohypertrophyofcalvesandquadriceps.
Dr.KhalidYusufElzohrySohagTeachingHospital2012 61

www.MRCPass.com

Neurology

NeurologyQ61
A 30 year old lady is found to have a left sided posterior communicating artery aneurysmoncerebralangiography. WhichoftheFollowingwouldyouexpecttofind?
A.Facialnervepalsy B.Leftpupillaryconstriction C.Sensorylosstotheleftsideoftheface D.Downgazepalsy E.Thirdnervepalsy

Answer:e)thirdnervepalsy. A posterior communicating artery aneurysm will cause compression of the third nerve, and therefore pupillary involvement from compression of the parasympathetic fibres thatrunontheoutsideofthethirdnerve.Thisleadstoadilatedpupil.Otherfeaturesof a third nerve palsy include ptosis, and a down and out eye. Upgaze and adduction is affected.
Dr.KhalidYusufElzohrySohagTeachingHospital2012 62

www w.MRCPass.c com

Ne eurology

Ne eurologyQ62
A 35 5 year old teacher t has s a right sid ded headache and blur rring of her r vision in the right eye. Shehaspreviously ha ad an episo odeofoptic cneuritis 3y years befor rehand, intheright ere was a right r affere ent pupillar ry defect an nd pale optic disc. eye. On examination, the The ere was wea akness of th he facial mu uscles on th he right. Tone and reflexes were brisk b on therightwithpower p of3/ /5inthearm mandthele eg.Shewas sapyrexial. t pa arietalmass s,whichwas sincomplet telyring MRIofthebrainrevealsa4cmlefttemporo hancing. The ere were also two sm mall white matter m lesio ons also vis sible in the e frontal enh area a. Wha atisthemo ostlikelydia agnosis?
A.Cereb brallymphom ma B.Multiplesclerosis s C.Acute edemyelinat tingencepha alomyelitis D.Lyme edisease E.Sarcoidosis

Answer:b)multiplesclero osis. The e episodes of o optic neu uritis are su uggestive of o MS. In view of the appearance e of the lesio on on MRI in association with wh hite matter changes, th he most like ely cause would w be ade emyelinatin nglesion. Larg gelesionssu uchasthesecancause eweakness orcranialnerve n defects.

WhitematterlesionsonMRI M seeninmultiplesc clerosis

Dr.Khalid K Yusuf fElzohrySo ohagTeachin ngHospital 2012 63

www.MRCPass.com

Neurology

NeurologyQ63
A 75 year old man has dysphasia and left sided arm weakness. He is known to have hypertension, asthma and rheumatoid arthritis. In addition he suffered from cluster headaches.Hesmokes20cigarettesaday. On examination, he has some weakness of the left hand. Visual fields, speech and sensationarenormal.Bloodpressureis190/90mmHg. WhichoftheFollowingismostlikely?
A.Partialposteriorcirculationinfarct B.Righthemispherelacunarinfarct C.Rightpontinehaemorrhage D.Totalanteriorcirculationinfarct E.Vertebrobasilarinsufficiency

Answer:b)righthemispherelacunarinfarct. The history suggests the dysarthriaclumsy hand syndrome, one of the classic lacunar syndromes thatare strokes inthe subcortical regions(orbrainstem) secondary to small vessel disease. The usual site of damage in the dysarthriaclumsy hand syndrome is the internalcapsuleorpons.
Dr.KhalidYusufElzohrySohagTeachingHospital2012 64

www.MRCPass.com

Neurology

NeurologyQ64
A 65 year old lady presents to the hospital with an ataxic gait. On examination, the patient had difficulty standing without assistance and minor movements of her head and body caused vertigo. There was mild postural tremor and marked hypotonia of the rightlimbs,mostlythearm. Finger to nose testing elicited marked intention tremor and disdiadochokinesis in the rightarm.Speechwasdysarthricbutcomprehensionwasgood. Whereisthelesionlikelytobe?
A.Rightpons B.Leftmedulla C.Rightcerebellarhemisphere D.Corpuscallosum E.Rightbasalganglia

Answer:c)rightcerebellarhemisphere. Lesions of the cerebellum (intention tremor, disdiadochokinesis) lead to motor signs ipsilateraltothelesion.
Dr.KhalidYusufElzohrySohagTeachingHospital2012 65

www.MRCPass.com

Neurology

NeurologyQ65
A 25 year old man is known to have epilepsy. He had a generalized tonic clonic seizures for15minutes. Whatdrugshouldbegiven?
A.IVphenytoin B.IVsodiumvalproate C.IVgabapentin D.IVlamotrigine E.IVlorazepam

Answer:E)IVlorazepam. Current consensus is that a benzodiazepine, notably lorazepam (Ativan), is the initial class of drug for the treatment of status epilepticus. A phenytoin, phenytoin sodium or fosphenytoinisthenextdrugtobeadministered.
Dr.KhalidYusufElzohrySohagTeachingHospital2012 66

www.MRCPass.com

Neurology

NeurologyQ66
A 28 year old woman is 30 weeks pregnant. She complains of a sudden onset generalisedheadache. On examination, she has pupils are which reactive bilaterally and there is a right sided thirdnervepalsy. Whichtestismostappropriate?
A.MRA B.MRV C.MRI D.CThead E.Lumbarpuncture

Answer:b)MRV. A headache in a pregnant patient, with associated cranial nerve palsy suggests cerebral venous sinus thrombosis. Treatment is with intravenous or low molecular weight heparin.
Dr.KhalidYusufElzohrySohagTeachingHospital2012 67

www.MRCPass.com

Neurology

NeurologyQ67
A45yearoldmanhassevereepisodesofdizzinesswithassociatedvomitingandpainin the right ear. This occurs once or twice a week. During these attacks he feels the surrounding environment spinning around. He also mentions a high pitched sound frequentlybeingpresent. On examination, during an attack, he has right horizontal nystagmus. Audiological testingrevealsrightsidedsensorineuraldeafness. Whatisthediagnosis?
A.Vestibularnystagmus B.Meniere'sdisease C.Benignparoxysmalpositionalvertigo D.Acousticneuroma E.Cerebellopontineangletumour

Answer:b)meniere'sdisease. Meniere's disease is caused by distension of the endolymphatic compartment of the inner ear. The symptoms ofMeniere's disease include vertigo, hearing loss and tinnitus. Thedizzinessisdescribedasaspinningorwhirlingfeelingandmaycauseproblemswith balance. Some people feel nauseated and vomit during an attack. Tinnitus refers to a ringingorroaringsoundintheear.Othersmaynoticesomehearingloss,especiallywith soundsthathavealowfrequency. Horizontal nystagmus is more commonly seen on examination, but vertical nystagmus mayalsooccur.
Dr.KhalidYusufElzohrySohagTeachingHospital2012 68

www.MRCPass.com

Neurology

NeurologyQ68
A 30 year old gynmast has sudden onset vertigo and dizziness. On examination, there is horizontal nystagmus, with a full range of eye movements. Her speech is slurred. There isintentiontremoranddisdiadochokinesiswhichisasymmetrical.Shehasanataxicgait. WhichoftheFollowinginvestigationswouldbemostappropriate?
A.CTofthehead B.MRIandMRAofheadandneck C.MRIwithenhancement D.Lumbarpuncture E.Otologicaltesting

Answer:b)MRIandMRAofheadandneck. The clinical picture is of an acute onset cerebellar syndrome, which suggest a vascular cause.Thiswouldinvolvetheposterior(vertebrobasilar)circulation.Ifheadacheorneck painwereassociated,avertebralarterydissectionwouldbemostimportanttoexclude. A Magnetic Resonance Angiography (MRA) will help to diagnose dissection, stenosis or thrombosis.
Dr.KhalidYusufElzohrySohagTeachingHospital2012 69

www w.MRCPass.c com

Ne eurology

Ne eurologyQ69
A30 0yearoldgym g instruct torcomplai inedofshoulderpainand a weakne esswhichhasbeen prog gressive ov ver the pas st 5 years. He is upse et about ha aving difficu ulty lifting weights whichhehadpreviously p been b ableto odosowith houtdifficul lty. On examinatio on, there wa as winging of the scap pula. Power r was reduc ced in the muscles arou und the sho oulder, with h bilateral wasting. w He also has so ome facial difficulty d rai ising his eyebrows.HisserumCKis400. atisthelike elydiagnosi is? Wha
A.Duchenne'smusc culardystrop phy B.Becke er'smusculardystrophy C.Myotonicdystrop phy D.Fascio oscapulohum meraldystro ophy E.Polym myalgiarheum matica

Answer:d)fasc cioscapulohumeral h dys strophy. Faci ioscapulohu umeral dys strophy (FSHD) is one of the mos st common types of muscular m dyst trophy.Itis sofautosom maldominantinheritan nce.Onsetis i usuallyag ge20years. . Initi ialweaknes ssisseenin nfacialmusc cles,startin ngintheorb bicularisoculi,orbicula arisoris, andzygomaticus. Sho oulderweak knessisthepresentingsymptominmoretha an82%ofpa atients.Win ngingof the scapula is the most characterist c tic sign. Cre eatine kinas se levels ar re raised. The drug Albu uterol whic ch relaxes bronchial s smooth mu uscle has been b shown n to increa ase lean mus sclemasswhen w usedover o aperiodofmonths.

Win ngingofthescapulainFSHD

Dr.Khalid K Yusuf fElzohrySo ohagTeachin ngHospital 2012 70

www.MRCPass.com

Neurology

NeurologyQ70
A 40 year old man presented with double vision and was found to have normal vertical eyemovements. On left lateral gaze, there was absence of adduction of the right eye, and nystagmus in theabductinglefteye. Thiseyemovementdisordercanbeexplainedbyalesioninthe:
A.Leftcerebellopontineangle B.Rightparietalarea C.Rightmediallongitudinalfasciculus D.Leftmediallongitudinalfasciculus E.Leftlateralmedulla

Answer:c)rightmediallongitudinalfasciculus. The diagnosis is right internuclear ophthalmoplegia due to a lesion in the right medial longitudinalfasciculus.Thelikelyunderlyingpathologyismultiplesclerosis,othercauses ofINOincludeagliomaorvascularlesion.
Dr.KhalidYusufElzohrySohagTeachingHospital2012 71

www.MRCPass.com

Neurology

NeurologyQ71
A 70 year old man presents with a history of falls. He has difficulty reading and walking down stairs. He has dysarthria, akinesia and rigidity. Power of the muscles is normal, reflexesarebrisk. Whatphysicalsignwillhelptoconfirmthediagnosis?
A.Gait B.Eyemovements C.Rombergssign D.Abdominalreflexes E.Plantarreflexes

Answer:b)Eyemovements. The patient has progressive supranuclear palsy parkinsonian features and gaze palsy. Demonstrationofimpairmentofvoluntarygazewillhelpconfirmthediagnosis.
Dr.KhalidYusufElzohrySohagTeachingHospital2012 72

www w.MRCPass.c com

Ne eurology

Ne eurologyQ72
A 55 5 year old man m has slo owly progre essive weak kness of his upper limb bs. On exam mination of the patient the physica al signs are wasting an nd weaknes ss of the sm mall muscles of the nd,flatteningofthemu usclesofthe eulnarbord derofthefo orearm. han The e upper limb b reflexes are a absent. Pain and temperature t e sensation n are reduced over the upper limb bs and uppe er chest whereas light touch and proprioception remain n intact. werlimbreflexesareex xaggeratedandplantarsareexten nsor. Low atisthelike elydiagnosi is? Wha
A.Norm malpressurehydrocephalus B.Multiplesclerosis s C.ArnoldChiarimalf formation D.Sever rekyphoscoliosis E.Kenne edy'ssyndrome

Answer:c)ArnoldChiarimalformatio m on. ArnoldChiari Malformati M ion is a con ndition in which the ce erebellum portion p of th he brain prot trudesintothespinalcanal. c Itma ayormayno otbeappar rentatbirth h. ArnoldChiari I type malfo ormation us sually cause es symptom ms in young adults and is often asso ociatedwith hsyringomy yelia,inwhichatubula arcavitydev velopswith hinthespina alcord. ArnoldChiariIItypemalfo ormationisassociatedwithmyelo omeningoce ele(adefec ctofthe spin ne) and hyd drocephalus s (increased d cerebrospinal fluid an nd pressure e within the e brain), which usually are apparent at birt th. The pat tient described has the features of an intramedullary ylesionofth hespinalco ord.

Chia ariMalform mation

Dr.Khalid K Yusuf fElzohrySo ohagTeachin ngHospital 2012 73

www w.MRCPass.c com

Ne eurology

Ne eurologyQ73
A 30 0 year old woman w has been diagn nosed with a cerebella ar cyst with MRI scanning. She is also a known n to have polycystic kidney dise ease. There e is a fam mily history of the condition. Wha atisthelike elydiagnosi is?
A.Phaeo ochromocyto oma B.VonHippel H Lindau usyndrome C.Hered ditaryhaemo orrhagictelangiectasia D.Multi iplesclerosis s E.Friedr riech'sataxia a

Answer:b)von nHippelLind dausyndrome. The e diagnosis is likely to o be von Hippel Lindau diseas se. There may m be cerebellar haemangioblas stomas, re etinal angio omas and polycystic c liver or kidneys. Ectopic eryt thropoietinsecretionb bythehaem mangioblast tomascause epolycythaemia.

Reti inalAngiom maseeninvon v HippelLindau L syndrome


Dr.Khalid K Yusuf fElzohrySo ohagTeachin ngHospital 2012 74

www w.MRCPass.c com

Ne eurology

Ne eurologyQ74
An 80 year old d man is admitted a with confusio on over the last 3 da ays. He has s a past med dical histor ry of hype ertension and a has ha ad frequen nt falls in the past. Clinical examinationisunremarka able. Whi ichofthese econditionsneedstobe eexcluded? ?
A.Intrac cranialhaem morrhage B.Subdu uralhaemato oma C.Meningitis D.Verte ebrobasilarst troke E.Encep phalitis

Answer:b)sub bduralhaem matoma. A patient p who o has had a fall ma ay have hit t his head d and deve eloped a subdural haematoma.Thiscanbee excludedby yaCTheadscan.

Righ htsidedsub bduralhaem matoma

Dr.Khalid K Yusuf fElzohrySo ohagTeachin ngHospital 2012 75

www.MRCPass.com

Neurology

NeurologyQ75
A 45 year old man presents with a sudden onset of headache in the posterior region, associatedwithvomiting. Neurological examination, including fundoscopy is unremarkable apart from slightly briskreflexes.Thereisnoneckstiffnessorphotophobia. WhichoftheFollowingmanagementoptionswouldbethemostappropriate?
A.CToftheheadandlumbarpuncture B.MRIofthehead C.CTofthehead D.SkullXray E.Dischargefromhospital

Answer:a)CToftheheadandlumbarpuncture. A subarachnoid haemorrhage (SAH) needs to be excluded. CT brain scan is normal in a third of patients with SAH. A lumbar puncture to look for xanthochromia in the CSF shouldthenbeperformed.
Dr.KhalidYusufElzohrySohagTeachingHospital2012 76

www.MRCPass.com

Neurology

NeurologyQ76
A 40 year old patient has presented with a generalised tonic clonic seizure for the first time.Thislastedfor10minutes. Whatadviceshouldbegivenregardingdrivingacar?
A.Nodrivingfor1month B.Nodrivingfor6months C.Nodrivingfor1year D.DrivingisallowedifEEGisnormal E.DrivingisallowedifCTscanisnormal

Answer:c)nodrivingfor1year. For a single seizure, driving is not permitted for 1 year. Also, a medical review is required before one is to do so and it is a requirement for the patient to inform the DriverandVehicleLicensingAuthority.
Dr.KhalidYusufElzohrySohagTeachingHospital2012 77

www.MRCPass.com

Neurology

NeurologyQ77
The sister of a patient who died from a subarachnoid hemorrhage due to a cerebral aneurysm is worried and is asking about her chances of having the same problem. She mentions that there family history of other deaths from subarachnoid haemorrhage. Sheishowever,asymptomatic. Whatshouldbedone?
A.Reassureandnothingelse B.CTscanofhead C.MRIheadscan D.Cerebralangiography E.Lumbarpuncture

Answer:C)MRIheadscan. This may be a case of familial subarachnoid haemorrhage. In those who have a first degreerelativewhogenuinelysufferedaSAH,theirriskofalsosufferingoneis37times that of the general population. MRI is better than CT scan for screening. Angiography is diagnosticbuttooinvasiveforscreening.
Dr.KhalidYusufElzohrySohagTeachingHospital2012 78

www w.MRCPass.c com

Ne eurology

Ne eurologyQ78
A45 5 yearold la ady hasa4week histo oryofpainand a difficultyseeing out of herrig ght eye. She has a visu ual acuity of o 6/18 in t the right an nd 6/6 in the t left. Th here is also a right erentpupilla arydefect. affe ichisthemostlikelyca ausefromth helistbelow w? Whi
A.Thyro oideyedisea ase B.Diabe eticretinopathy C.Astrocytoma D.Multi iplesclerosis s E.Retini itispigmento osa

Answer:d)multiplesclero osis. Opt ticneuritissecondary s tomultiplesclerosis s canpresentin nthismann ner.

Disc cpallor(opt ticneuritis)

Dr.Khalid K Yusuf fElzohrySo ohagTeachin ngHospital 2012 79

www.MRCPass.com

Neurology

NeurologyQ79
A40yearoldladypresentswithdroopingofhereyelidsanddoublevision.Shedoesnot have proptosis. There is no muscle wasting around the face. She has diplopia on downgaze during examination and also proximal muscle weakness of her upper limbs. Myastheniagravisisdiagnosed. Whichdrugismostlikelytoimprovehersymptoms?
A.Betainterferon B.Intravenousimmunoglobulin C.Benztropine D.Pyridostigmine E.Bromocriptine

Answer:d)pyridostigmine. The condition described is Mysthenia Gravis rather than Grave's eye disease or Myotonic dystrophy (frontal balding). Pyridostigmine is an anticholinesterase which reduces acetylcholine breakdown and hence improve symptoms of fatiguability in myastheniagravis.
Dr.KhalidYusufElzohrySohagTeachingHospital2012 80

www.MRCPass.com

Neurology

NeurologyQ80
A 40 year old bank clerk presents with a headache, nausea and ptosis of the left eye with blurred vision. She does not have fatiguability of her eye movements. Examination revealed sw elling on the left side of face, proptosis and chemosis of the left eye, left mastoidswellingandleftophthalmoplegiainvolvingcranialnerves. Thepupilsizeswereequal.ACTofherheadisnormal. Whatisthelikelydiagnosis?
A.Thirdnervepalsy B.Myastheniagravis C.Pituitarytumour D.Cavernoussinusthrombosis E.Horner'ssyndrome

Answer:d)cavernoussinusthrombosis. A history of headache and no other obvious cause of ptosis is suggestive of cavernous sinus thrombosis. Third nerve palsy is associated with dilated pupil and Horner's syndromeisassociatedwithmiosis.ACTcanbenormal,anddiagnosisisconfirmedwith MRI.
Dr.KhalidYusufElzohrySohagTeachingHospital2012 81

www.MRCPass.com

Neurology

NeurologyQ81
A 65 year old man presents with an episode of amnesia for the second time. 2 days ago he had an episode of confusion, according to his wife. He was, how ever, able to have a normal conversation despite having been found wandering. After 2 hours, he abruptly returnedtonormalandcouldnotrememberwhathappened. Whatisthemostlikelydiagnosis?
A.Alcoholicencephalopathy B.Subarachnoidhaemorrhage C.Complexpartialseizure D.Transientischaemicattack E.Transientglobalamnesia

Answer:E)tranbsientglobalamnesia. Transient global amnesia (TGA) is a temporary and isolated disorder of memory which maylastseveralhours.Heavyexerciseandthecoldareknownprecipitants.
Dr.KhalidYusufElzohrySohagTeachingHospital2012 82

www.MRCPass.com

Neurology

NeurologyQ82
A40yearoldmanpresentswithfingerweaknesswhichwasdiagnosedasanulnarnerve lesion. WhichoftheFollowingmusclesissuppliedbytheulnarnerve?
A.Interossei B.Lateraltwolumbricals C.Opponenspollicis D.Abductorpollicisbrevis E.Flexorpollicisbrevis

Answer:a)interossei. The interossei muscles and medial two lumbricals are supplied by the ulnar nerve. The lateral two lumbricals (anatomical position), opponens pollicis, abductor pollicis brevis andflexorpollicisbrevisaresuppliedbythemediannerve.
Dr.KhalidYusufElzohrySohagTeachingHospital2012 83

www.MRCPass.com

Neurology

NeurologyQ83
A 35 year old alcoholic presents with unsteadiness whilst walking. On examination he has increased tone and brisk reflexes in the right leg. Proprioception is abnormal in the right leg. There is loss of vibration sense in the right leg. There is decrease in pain and temperaturesensationintheleftleg. Whichoneofthefollowingconditionsismostlikelytoberesponsibleforhisweakness?
A.Syringomyelia B.Subacutecombineddegenerationofcord C.Friedrich'sataxia D.GuillainBarresyndrome E.BrownSequardsyndrome

Answer:e)BrownSequardsyndrome. Brown Sequard syndrome which describes hemisection of the spinal cord, causes ipsilateral UMN signs and proprioception loss (corticospinal tract and dorsal column decussate at the medulla), and contralateral sensory loss in pain and temprature (the spinothalamic tracts decussate at the same level). The rest of the conditions (syringomyelia, subacute degeneration of cord, Friedrich's ataxia) can cause cerebellar signsorpatchysensorylossbutshouldbebilateral.
Dr.KhalidYusufElzohrySohagTeachingHospital2012 84

www w.MRCPass.c com

Ne eurology

Ne eurologyQ84
A 45 5 year old man has bilateral ptos sis. He men ntions a pas st history of cataracts, , frontal bald ding and weakness w of f the facial muscles. On O examina ation, he ha as a firm gr rip with diffi icultyrelaxing. atisthedia agnosis? Wha
A.Multi iplesclerosis s B.Moto orneurondis sease C.Parkin nson'sdiseas se D.Derm matomyositis E.Myoto onicdystrop phy

Answer:e)myo otonicdystr rophy. Myo otonic dyst trophy is au utosomal do ominant. It is a trinucl leotide repe eat disorde er which exhibits anticipation (wo orse with successive generation ns). Associa ated features are cata aracts,diabetes,testicu ularatrophyandcardia acconducti ionabnorm malities.

Myo otonicDystrophy
Dr.Khalid K Yusuf fElzohrySo ohagTeachin ngHospital 2012 85

www.MRCPass.com

Neurology

NeurologyQ85
A50yearoldalcoholicisadmittedtoA+Ewithunsteadinessandconfusion.BMis7. Whichisthemostappropriatetreatment?
A.Glucose B.Lorazepam C.IVthiamine D.IVvitaminB12 E.IVvitaminK

Answer:c)IVthiamine. This patient is likely to have Wernicke's encephalopathy causing confusion. IV thiamine shouldgiventoreducetheprogression.ThisiscontainedinPabrinex.
Dr.KhalidYusufElzohrySohagTeachingHospital2012 86

www.MRCPass.com

Neurology

NeurologyQ86
A man presents with generalised weakness. On examination, fatiguability was demonstrated.AdiagnosisofEatonLambertsyndromewasmade. Whatformofantibodyisfoundinthiscondition?
A.AntiPurkinje B.Anticholinesterase C.Neuromuscularjunction D.Potassiumchannels E.Voltagegatedcalciumchannels

Answer:E)voltagegatedcalciumchannels Eaton Lambert syndrome is frequently associated with a malignancy e.g. bronchial. The disorderisassociatedwithantibodiesagainstvoltagegatedcalciumchannels.
Dr.KhalidYusufElzohrySohagTeachingHospital2012 87

www w.MRCPass.c com

Ne eurology

Ne eurologyQ87
A 20 2 year old d man injured himself whilst sn now boarding. On exa amination, he has wea aknessofelbowflexion nandlossof o sensationoverthera adialaspect tofherfore earm. Whi ichoftheFo ollowingne ervesisdam maged?
A.Musc culocutaneou usnerve B.Brach hioradialisne erve C.Radialnerve D.Mediannerve E.Ulnarnerve

Answer:a)mus sculocutane eousnerve. . The e fibers of the t musculocutaneous nerve ori iginate in the lower cervical c spin nal cord (usu ually C5 to C7), travel via the late eral cord of f the brachi ial plexus, and a supply sensory and motor inn nervation to o the upper arm, elbo ow , and fo orearm. It supplies s the e biceps whichcontrolselbowflexion.Sensati ionistothe elateralare ea(lateralcutaneousnerve)of theforearm.

Dr.Khalid K Yusuf fElzohrySo ohagTeachin ngHospital 2012 88

www.MRCPass.com

Neurology

NeurologyQ88
An 18 year old woman is referred to the GP by her teacher. The teacher was concerned about frequent episodes of day dreaming during class and poor examination results overthepastyear. WhichoneoftheFollowingneedstobeexcluded?
A.Vasovasagalsyncope B.Pseudoseizure C.Anxietydisorder D.Visualhallucinations E.Absenceseizures

Answer:E)Absenceseizures. Daydreaming in children can be easily confused with absence or complex partial seizures, in which staring is a prominent and common feature. However, lip smacking, eye blinking, or stiffening of muscle groups is common during seizures but not during daydreaming.
Dr.KhalidYusufElzohrySohagTeachingHospital2012 89

www.MRCPass.com

Neurology

NeurologyQ89
A 20 year old man presents with a 6 month history of depression and painful sensory disturbance in both legs. He has also become very confused. There are myoclonic jerks observedinhislegs.HisMRIscanrevealsthalamichyperintensityandEEGisnormal. Themostlikelydiagnosisis:
A.NewvariantCJD B.Huntington'sdisease C.Wilson'sdisease D.Progressivemultifocalleucoencephalopathy E.Paraneoplasticsyndrome

Answer:a)newvariantCJD. New variant CJD commonly presents in young adults painful sensory symptoms in the lower limbs and also psychiatric symptoms. Cognitive impairment, pyramidal signs, myoclonusandprimitivereflexesthendevelop. MRI commonly shows high signal on T2w eighted images in the pulvinar (posterior aspect of thalamus). EEG is often normal, unlike sporadic CJD, in which triphasic waves areobserved.
Dr.KhalidYusufElzohrySohagTeachingHospital2012 90

www.MRCPass.com

Neurology

NeurologyQ90
A 60 year old woman is admitted with a severe headache. CT scan conforms a subarachnoidhaemorrhage.Sheinitiallymakessatisfactoryprogressbut7dayslaterher levelofconsciousnessbeginstodeteriorate. Themostlikelycauseofthedeteriorationis:
A.Cerebraloedema B.Coningofthemedulla C.Meningitis D.Encephalitis E.Acutehydrocephalus

Answer:e)acutehydrocephalus. Organised blood in the subarachnoid space may cause obstruction to the flow of cerebrospinal fluid (impaired absorption in the arachnoid villi). 10% of patients will requireCSFdiversionorshunting.
Dr.KhalidYusufElzohrySohagTeachingHospital2012 91

www.MRCPass.com

Neurology

NeurologyQ91
A 62 year old woman has several episodes of dizziness particularly when she turns her head. 2 months ago, she had an attack of vertigo, without deafness or tinnitus, lasting for a few minutes. Over the last month, she had five further attacks of vertigo, accompanied by moderate headache and a left homonymous hemianopia, lasting for about a quarter of an hour. After the last episode she developed persistent unsteadinessofgait,andwasadmittedtohospital. Onexaminationtherewasnormalvisualfields.werefull.Therewasrhythmichorizontal nystagmus, slight weakness of the right external rectus muscle without diplopia, and ataxiaofgait,provokedbyturning.Thebloodpressurewas160/80mmHg. Whatisthediagnosis?
A.ParietallobeCVA B.FrontallobeCVA C.Vertebrobasilarinsufficiency D.Syringomyelia E.Brownsequardsyndrome

Answer:c)vertebrobasilarinsufficiency. Vertebrobasilar (posterior) circulation constitutes the arterial supply to the brain stem, cerebellum, and occipital cortex. Bilateral visual loss, dizziness, speech disturbances, drop attacks and transient global amnesia are features of vertebrobasilar insufficiency. MRI/MRAaregoodinvestigationstoinvestigateforvertebralorbasilararterialdisease.
Dr.KhalidYusufElzohrySohagTeachingHospital2012 92

www.MRCPass.com

Neurology

NeurologyQ92
A55yearoldmanhasdevelopedweaknessoverthepast3weekswhichhasaffectedhis walking. He has no significant past medical history. On examination, he had decreased sensationperipherallyinthelegsandalsoflaccidreflexesintheankles.ACTofthehead wasnormalandlumbarpuncturewasdone. Resultswere: protein0.75(<0.43g/l) glucose4(3.3to4.4mmol/l) lymphocytes7(<5/mm3) Whatisthelikelydiagnosis?
A.Multiplesclerosis B.GuillainBarresyndrome C.Lymphocyticmeningitis D.Tuberculousmeningitis E.Syringomyelia

Answer:B)GuillainBarresyndrome. In Guillain Barre syndrome, CSF protein is elevated in most patients after the second or third week of illness. The gamma globulin fraction is usually raised. Cells, usually monocytic,arefoundin20%ofcases.
Dr.KhalidYusufElzohrySohagTeachingHospital2012 93

www w.MRCPass.c com

Ne eurology

Ne eurologyQ93
A 65 5 year old patient has s progressiv ve dementia a. His wife mentions that t he has urinary inco ontinenceandanataxicgait. Wha atishisCTscan s likelyto t show?
A.Paras sagittalmass s B.Multipleinfarcts C.Largeventricles D.Cereb bellartumou ur E.Berryaneurysm

Answer:c)larg geventricles s. The e diagnosis is normal pressure hy ydrocephalus. Dement tia, urinary y incontinen nce and unst teady gait are seen. Typically T there is no papilloedem p ma. There are a large ve entricles caused by com mmunicating g hydrocep phalus. Ventricular shu unting may y help improve the sym mptoms.

Nor rmalPressureHydrocephalus
Dr.Khalid K Yusuf fElzohrySo ohagTeachin ngHospital 2012 94

www.MRCPass.com

Neurology

NeurologyQ94
A 60 year old lady presents with acute onset unsteadiness and dizziness. Neurological examinationshowsarightsidedHornerssyndromeandnystagmus.Thereisalsolossof painandtemperaturesensationontheleftsideofthetrunkandintheleftarmandleg. Hergaitisataxic. Whichisthecorrectdiagnosis?
A.Posteriorinferiorcerebellararteryocclusion B.Medullaryinfarct C.Posteriorcerebralarteryocclusion D.Middlecerebralarteryocclusion E.Posteriorcommunicatingarteryhaemorrhage

Answer:a)posteriorinferiorcerebellararteryocclusion. There are a complex of symptoms caused by occlusion of the posterior inferior cerebellar artery or one of its branches supplying the lower portion of the brain stem, resulting in sensory and sympathetic disturbances, cerebellar and pyramidal tract signs, and evidence of partial involvement of the fifth, ninth, tenth, and eleventh cranial nerves. Onset isusuallyacutewithsevere vertigo.Nausea, vomiting,ipsilateralataxia,muscular hypertonicity, pastpointing and other cerebellar signs are often present. Horner's syndrome is usually present. Sensory disturbances include ipsilateral loss of pain and temperature perception of the face and contralateral hypoesthaesia for pain and temperatureofthetrunkandextremities. Theaffectedpersonshavedifficultyinswallowing.Personswellover40yearsofageare mostoftenaffected.
Dr.KhalidYusufElzohrySohagTeachingHospital2012 95

www w.MRCPass.c com

Ne eurology

Ne eurologyQ95
A 60 0 year old man m has ne ew onset re eceptive and d expressiv ve dysphasia a. His past medical historyinclude esdiabetesa andhyperte ension. ncreased to one and exte ensor plant tar reflex on n the right. He also On examination, he hasin hasweaknessof o therightlegwithsensorylossover o thesam meside. Whi ichvascular rlesionislik kely?
A.Anter riorcerebralartery B.Super riormiddlec cerebralartery C.Inferiormiddlece erebralartery y D.Poste eriorcerebra alartery E.Posteriorinferiorcerebellarartery

Answer:a)ante eriorcerebr ralartery. A middle m cere ebral artery y occlusion is likely to o cause tota al hemipleg gia. In view w of the part tialweakne ess,ananteriorcerebra alartery(ACA)lesionismorelikely. Com mplete infar rction due to t occlusion of one AC CA distal to o the anterior communicating arte eryresultsinasensory ymotordef ficitoftheopposite o foo otandlegand a alesser rdegree of paresis p of the t arm wi ith sparing of the fac ce. Foot dro op is a com mmon finding and diffi icultyinwalkingisevengreaterth hanexpecte edfromthe eweakness present. Dysphasia could occur with the occlu usion of a branch b of the left ACA A. Branch oc cclusion of the ACA can n cause only y parts of th he total syn ndrome, pro oducing a sp pastic weak kness or cort ticalsensory ylossinthe eoppositef footandleg g.

AnteriorCereb bralArteryIn nfarct


Dr.Khalid K Yusuf fElzohrySo ohagTeachin ngHospital 2012 96

www.MRCPass.com

Neurology

NeurologyQ96
A 35 year old man is admitted with acute right sided weakness and slurred speech. He does not have ahistory of hypertension,diabetes or high cholesterol.Thereis nofamily history of CVA. He does not smoke. He complains of headaches and generalised limb weaknessinfrequently. Onexamination,hehashypotoniaandweaknessintheproximalmusclesmorethanthe distalmuscles.Hislaboratorytestsshowahighlactatetopyruvateratio. Whatisthelikelydiagnosis?
A.Polymyositis B.Inclusionbodymyositis C.MELAS D.Polymorphonuclearleukoencephalopathy E.Neuroacanthocytosis

Answer:c)MELAS. This patient has MELAS (myopathy, encephalopathy, lactic acidosis and stroke like episodes). Lacticacidosisisaveryimportantfeatureofthisdisorder,asmeasuredbyahighlactate topyruvateratio. How ever, in general, lactic acidosis does not lead to systemic metabolic acidosis, and it maybeabsentinpatientswithimpressiveinvolvementofthecentralnervoussystem. Patients have a myopathy causing proximal muscle weakness and hypotonia, seizures andstrokelikeepisodes.Itisamitochondrialinheriteddisorder.
Dr.KhalidYusufElzohrySohagTeachingHospital2012 97

www.MRCPass.com

Neurology

NeurologyQ97
A 75 year old lady complains of a headache for 2 days in the right side of the head. The pain is worse when she is chewing or talking. She has also had mild fevers and sw eats. HerESRis80mm/hr.Thevisionintherighteyeis6/18andherlefteyeis6/6. Whatisthebestcourseofaction?
A.Startivmethylprednisolone B.Organiseandawaittemporalarterybiopsy C.CToftheheadtoexcludespaceoccupyinglesion D.MRIofthebrain E.Refertoanophthalmologist

Answer:a)startivmethylprednisolone. Thereareearlysigns of visual loss sohighdosesteroids should be commenced withthe suspicion of temporal arteritis. Although all the other options are reasonable, they may taketimeandthereshouldnotbedelayincommencingsteroids.
Dr.KhalidYusufElzohrySohagTeachingHospital2012 98

www w.MRCPass.c com

Ne eurology

Ne eurologyQ98
A 35 year old woman pre esents with h double vis sion that is worst whe en trying to o read a boo okorwalkdownstairs. Themostlikely ydiagnosisis: i
A.Progr ressivesupra anuclearpals sy B.4thnervepalsy C.3rdne ervepalsy D.6thnervepalsy E.Intern nuclearophthalmoplegia a.

Answer:b)4thnervepalsy y. The e superior oblique muscle is innervated by th he fourth ne erve. The action a is to depress the eye and is maximally effective when w the ey ye is looking medially, hence diplopia on read dingorgoin ngdownstai irsistypicaloffourthnerve n paraly ysis.

Dr.Khalid K Yusuf fElzohrySo ohagTeachin ngHospital 2012 99

www.MRCPass.com

Neurology

NeurologyQ99
A 65 year old man has muscle weakness, particularly around the thighs and shoulders. HisCKis2,200U/l.EMGshowsreducedamplitudeanddurationofmotorunits. Whatisthelikelydiagnosis?
A.Dermatomyositis B.Myastheniagravis C.Myotonicdystrophy D.Peripheralneuropathy E.Multiplesclerosis

Answer:a)dermatomyositis. These EMG changes are consistent with a myositis. In motor neuron disease, fibrillation isseen.Inmyasthenia,thereisdiminishedresponsetorepetitivestimulation.
Dr.KhalidYusufElzohrySohagTeachingHospital2012 100

www.MRCPass.com

Neurology

NeurologyQ100
A 42 year old man presents to A+E complaining of severe lower back pain Following carpentrywork.Thepainradiatestohisleftbuttockandthigh. Onexamination,hewasabletostraightlegraiseto45degreesonlyontheleftside.The sciatic stretch test is positive. He has difficulty plantar flexing his left ankle and has abnormalsensationontheplantaraspectofthefoot. Whatisthediagnosis?
A.Caudaequinasyndrome B.L2/L3discprolapse C.L4/L5discprolapse D.L5/S1discprolapse E.Commonperonealnerveinjury

Answer:D)L5/S1disease. AnkledorsiflexionisgenerallysuppliedbyL4/L5andplantarflexionsuppliedbyS1/S2. thiscaseislikelytobeduetosciaticnervepalsy.


Dr.KhalidYusufElzohrySohagTeachingHospital2012 101

www.MRCPass.com

Neurology

NeurologyQ101
A 60 year old man is brought to hospital having collapsed to the ground suddenly and was unable to move his left leg or arm. There was no loss of consciousness. He has a past medical history of hypertension only. The episode lasted a few seconds and he has beenrelativelywell.Examinationrevealsamildhemiparesisoftheleftarmandleg. Whichisthelikelydiagnosis?
A.Pontinehaemorrhage B.Primaryepilepsy C.Medullaryhaemorrhage D.Rightinternalcapsuleinfarct E.Leftinternalcapsuleinfarct

Answer:d)rightinternalcapsuleinfarct. This patient is likely to have a lacunar infarct involving the internal capsule, causing transientcontralateralhemiparesis.
Dr.KhalidYusufElzohrySohagTeachingHospital2012 102

www.MRCPass.com

Neurology

NeurologyQ102
A25yearoldmanhashadbehaviouraldisturbancerecently.Hisparentsmentionedthat his brother has been investigated for liver problems recently. On examination, he has a MMSEscoreof28/30.Hehasamasklikefaceandwasnoticedtohavehypersalivation. Wheninvestigationsarecomplete,whichdrugismostlikelytobeusedfortreatment?
A.Desferrioxamine B.Cocareldopa C.Penicillamine D.Interferonalpha E.Chlorpromazine

Answer:C)penicillamine. The likely diagnosis is Wilsons disease. Most patients who present with neuropsychiatric manifestations have cirrhosis. The most common presenting neurologic feature is asymmetric tremor, occurring in approximately half of individuals with Wilson disease. Frequent early symptoms include difficulty speaking, excessive salivation, ataxia, masklike facies, clumsiness with the hands, and personality changes. Thediseaseisautosomalrecessive.Penicillamineisusedasacopperchelator.
Dr.KhalidYusufElzohrySohagTeachingHospital2012 103

www.MRCPass.com

Neurology

NeurologyQ103
A 55 year old man presents with a history of slowly progressive, abnormal movements of his body. On examination there are jerky, semipurposive movements involving the entirebodyandabnormaltonguemovements. Whatisthelikelydiagnosis?
A.Parkinson'sdisease B.Motorneurondisease C.Frontallobetumour D.Cervicalspondylosis E.Huntington'sdisease

Answer:e)Huntington'sdisease. Therearemanycausesofchorea. InheritedAtaxiatelangiectasia,Huntingtondisease,Wilsondisease. Drugs Anticonvulsants (eg, phenytoin, carbamazepine, phenobarbital), Antidopaminergicagents(eg,phenothiazines,haloperidol,metoclopramide) Behet disease, antiphospholipid antibody syndrome, Bacterial endocarditis, Herpes simplexencephalitis,Lymedisease.
Dr.KhalidYusufElzohrySohagTeachingHospital2012 104

www.MRCPass.com

Neurology

NeurologyQ104
A 40 year old man has a 5year history of rightsided cluster headaches with recurrent rightsidedheadacheswhichlastfor2hours.Thereisapatternofdailyoccurrencefor2 to 3 weeks, followed by a month or so without headaches. The headaches were describedasexplosiveandweregraded9to10inseverityonavisualanalogscaleof1 to10. The quality of the head pain was reported to be throbbing, sharp, shooting, as well as aching. The pain was localized around the right eye, behind the right ear, and in the occipital region. There are no associated visual symptoms, but occasionally flushing is associated.Hegetstheseheadachesfrequentlyduringthewinter. Whatisthelikelydiagnosis?
A.Migraine B.Trigeminalneuralgia C.Absenceseizures D.Sagittalsinusthrombosis E.Clusterheadache

Answer:e)Clusterheadache. Inclusterheadaches,thereisanassociationwithautonomicfeatures,particularlymiosis and ptosis. Cluster headaches occur during the same months in the year typically and are almost five times more common in males. A opthalmic division of trigeminal nerve distributioninvolvementisalsocommon.
Dr.KhalidYusufElzohrySohagTeachingHospital2012 105

www.MRCPass.com

Neurology

NeurologyQ105
A 30 year old man complains of spasms in his neck. He has, over the past three years, noticedacrampysensationinhisneckassociatedwithcontractionofthemuscleonthe left. This has now got frequent and uncontrollable. Neurological examination is normal, butontappingtheleftsideoftheneckmuscle,itcontractsandspasmsoccur. WhichoftheFollowingmedicationmayhelpthis?
A.Methysergide B.Levodopa C.Phenytoin D.Lamotrigine E.Methyldopa

Answer:b)levodopa. This patient has a dystonia in the neck. Other examples are blepharospasm and torticollis. This could be helped by levodopa or diazepam. In severe cases, botulinum injections mayalsohelp.
Dr.KhalidYusufElzohrySohagTeachingHospital2012 106

www.MRCPass.com

Neurology

NeurologyQ106
A45yearoldmanwasreferredforassessmentofunsteadygait,whichhasbeenpresent for6months.Hehaslostastoneinweightovertheseveralmonths.AnMRIofthebrain showsmultiplehighsignalareas. A biopsy was taken of one of the lesions. The report shows perivascular infiltrates of lymphocytes affecting white and gray matter. There was minimal myelin loss. CSF examinationshowsraisedwhitecellcount,andoligoclonalbandswerenotraised. Whatisthemostlikelydiagnosis?
A.CreutzfeldtJakobdisease B.Progressivemultifocalleucoencephalopathy C.CNSlymphoma D.Glioma E.HSVencephalitis

Answer:c)CNSlymphoma. Thewhitematterlesionssuggesteithermultiplesclerosisorlymphoma.Infiltrationwith lymphocytes in this type of presentation makes a primary lymphoma of the central nervoussystemlikely.
Dr.KhalidYusufElzohrySohagTeachingHospital2012 107

www.MRCPass.com

Neurology

NeurologyQ107
A 30 year old man has a painful right eye. On examination, there is decreased visual acuityandarelativeafferentpupillarydefectoftherighteye. Whatisthediagnosis?
A.Opticatrophy B.Opticneuritis C.Glaucoma D.Retinitispigmentosa E.Internuclearopthalmoplegia

Answer:b)opticneuritis. A painful eye with loss of vision, and also RAPD suggests optic neuritis. The most likely underlyingcauseismultiplesclerosis.
Dr.KhalidYusufElzohrySohagTeachingHospital2012 108

www.MRCPass.com

Neurology

NeurologyQ108
A 50 year old man has developed unsteadiness. He has hypertension and is a smoker of 20/day. On examination he has a cerebellar ataxia and pass pointing. His CXR shows a righthilarmass. WhichoftheFollowingismostlikelytorevealthediagnosis?
A.AntiGM1antibodies B.AntiYoantibodies C.Phytanicacidlevels D.Serumcopper E.Serumferritin

Answer:b)AntiYoantibodies. The most likely diagnosis is a paraneoplastic syndrome. Anti Yo antibodies are found in around half of all patients with paraneoplastic cerebellar degeneration. Associated with small cell carcinoma, ovarian tumours and Hodgkins lymphoma. Anti Hu antibodies are associated with small cell carcinoma of the lung. It is usually associated with sensory neuropathyorwithencephalomyelitis.
Dr.KhalidYusufElzohrySohagTeachingHospital2012 109

www w.MRCPass.c com

Ne eurology

Ne eurologyQ109
A 65 year old man has early signs o of cognitive e impairmen nt. His wife e describes urinary inco ontinence and a unstead diness on walking w in th he past six months. Ex xamination reveals atax xiaandhehas h anMTSscoreof7/ /10. atisthemo ostlikelydia agnosis? Wha
A.Lewybodydiseas se B.Alzhe eimer'sdeme entia C.Creut tzfeldtjakobdisease D.Norm malpressurehydrocephalus E.Shydragersyndro ome

Answer:d)nor rmalpressurehydrocep phalus. Nor rmal pressu ure hydroce ephalus (NP PH) is a clin nical sympto om comple ex character rized by abn normal gait, urinary inc continence, and dementia. Ventri icular enlarg gement occ curs out of proportion p on the CT T scan. Surgical CSF shunting remains r the e main tre eatment mod dality.

Dr.Khalid K Yusuf fElzohrySo ohagTeachin ngHospital 2012 110

www.MRCPass.com

Neurology

NeurologyQ110
A40yearoldmanhasa4weekhistoryofdizziness,andvomitting.Theonsetwasacute. He feels that the world was spinning and his balance is poor. His hearing is normal. Thereisnofamiliyhistory.Onexamination,hehasnocerebellarsigns. Whatisthemostlikelydiagnosis?
A.Benignpositionalvertigo B.Friedrich'sataxia C.Acousticneuroma D.Vestibularneuronitis E.Vertebrobasilarcirculationinsufficiency

Answer:d)vestibularneuronitis. Vestibular neuronitis often presents with acute vestibular disturbance that gradually resolves over a few weeks. It is associated with viral infections. Treatment is conservativeandsupportive.
Dr.KhalidYusufElzohrySohagTeachingHospital2012 111

www.MRCPass.com

Neurology

NeurologyQ111
A 62 year old man complains of headache and on examination of his visual fields you detectarightupperquadrantopia. Whereisthelesion?
A.Opticchiasm B.Righttemporallobe C.Rightparietallobe D.Leftparietallobe E.Lefttemporallobe

Answer:e)Lefttemporallobe. Upper quadrantopia is due to temporal lobe lesions and lower quadrantopia to parietal lobelesions.Thevisualfieldiscausedbythecontralaterallesion.
Dr.KhalidYusufElzohrySohagTeachingHospital2012 112

www w.MRCPass.c com

Ne eurology

Ne eurologyQ112
A 65 6 year old d man pre esents with muscle weakness w and difficult ty swallowing. On examination, he h has pro oximal and distal upp per and low wer limb weakness. w There T is sting of the e intrinsic muscles m of t the fingers and of his thigh musc cles. CK is elevated e was andEMGfindin ngsareconsistentwith hamyopath hicprocess. . ichconditio onislikely? Whi
A.Polym myositis B.Derm matomyositis C.Inclus sionbodymy yositis D.Moto orneurondis sease E.Duche ennemuscul lardystrophy y

Answer:c)inclusionbodymyositis. The edistributionofthemu usclesinvolv vedaretyp picalofinclu usionbody myositis,although the distributio on is usually asymme etric. In this condition n, dysphagi ia and respiratory invo olvement can c also oc ccur. Musc cle biopsy shows intr racellular in nclusions (a amyloid prec cursorprotein,ubiquit tins)andinf flammatory yinfiltrates.

InclusionbodyinIBM

Dr.Khalid K Yusuf fElzohrySo ohagTeachin ngHospital 2012 113

www w.MRCPass.c com

Ne eurology

Ne eurologyQ113
A65 5 year oldfemale f isad dmittedwith a historyofsevere headaches. h The T headac ches are iated with nauusea. wor rse in the morning m and d are associ n Sh he mention ns a blackou ut a few wee ekspriortoadmission. On examinatio on of the patients p fu undus, the optic cups s were fille ed and the medial rginsofthediscswereblurred. mar ichisthemostlikelyca auseofthispatientsco ondition? Whi
A.Cereb bellartumou ur B.Front tallobetumo our C.Temp porallobeep pilepsy D.Benig gnintracranialhypertens sion E.Migra aine

Answer:b)fron ntallobetumour. The e history of headaches which are worse in th he morning g and accom mpanied by nausea sugg gests increa ased intrac cranial pressure. This is confirme ed by the fa act the opt tic discs have blurred margins. m Th he history o of a probab ble seizure with w collaps se, in comb bination with halltheoth herfactorssuggests s alikelyfronta allobetumo our.

Fron ntalLobeTu umour

Dr.Khalid K Yusuf fElzohrySo ohagTeachin ngHospital 2012 114

www.MRCPass.com

Neurology

NeurologyQ114
A 40 year old woman presents with gradual onset severe headache and visual blurring. Onexamination,shehasbilateralpapilloedema. WhichoftheFollowingmayhelpimprovethecondition?
A.VitaminA B.Prednisolone C.Minocycline D.Cyclosporin E.Acetazolamide

Answer:e)acetazolamide. Vitamin A, prednisolone, minocycline and cyclosporin make idiopathic intracranial hypertension (used to be known as benign intracranial hypertension worse. Acetazolamide is used to treat IIH. IIH is likely to be due to high pressure caused by the builduporpoorabsorptionofcerebrospinalfluidinthesubarachnoidspacesurrounding the brain. The disorder is most common in women between the ages of 20 and 50. Symptomsincludeheadache,nausea,vomiting,andpulsatingintracranialnoises,closely mimickingsymptomsofbraintumors.
Dr.KhalidYusufElzohrySohagTeachingHospital2012 115

www.MRCPass.com

Neurology

NeurologyQ115
A 35 year old woman has been admitted to hospital for investigation of progressive weakness in her legs. For the past 5 years. The patient's mother has similar difficulties withweaknessandsensoryproblems. Examination revealed power of 3/5 distally in the upper and lower limbs with a glove andstockingpatternsensorylosstopainandtouch. Whatisthelikelydiagnosis?
A.Hereditaryneuropathywithliabilitytopressurepalsies B.Friedrich'sataxia C.Chronicinflammatorydemyelinatingpolyneuropathy D.Multiplesclerosis E.Hereditarysensorimotorneuropathy

Answer:e)hereditarysensorimotorneuropathy. In view of the family history, this patient is most likely to have hereditary sensori motor neuropathytypeI(CharcotMarieToothdisease). HMSN 1 is the most common form of hereditary neuropathy. Severely and uniformly slowed nerve conduction velocities (NCVs) and primary hypertrophic myelin pathology with prominent onion bulbs and secondary axonal changes are the hallmarks of the disease. Motor symptoms predominate over sensory symptoms. Often, patients report loss of balance, muscle weakness, and foot deformities. Onset in the first decade of life istypical,butdiseasedevelopsinsomepatientsinyoungormidadulthood. HMSN 2, on the other hand, represents the nondemyelinating neuronal type with relativelynormalNCVsandprimary axonalpathology.Althoughnervesarenotenlarged in the neuronal form, weakness often is less marked and onset of this neuropathy is delayed. Peripheral nerves are not enlarged clinically, and weakness of feet and leg muscles predominates; hands are less severely affected than the legs. Patients experience sensory loss in the distal extremities, and foot deformities (ie, pes cavus) tendtobelessmarkedthanthoseofHMSN1.
Dr.KhalidYusufElzohrySohagTeachingHospital2012 116

www.MRCPass.com

Neurology

NeurologyQ116
An 80 year old lady is admitted complaining of acute onset weakness in both legs and low back pain for the previous 4 weeks. She has lost 5 kg of weight recently, and has a Hb of 8.5 g/dl. Her cranial nerves and upper limbs were normal but she has reduced powerinbothlowerlimbsof3/5.ShehasasensorylevelatT12andplantarreflexesare upgoing.Rectaltonewasnormal. Whichisthebestnextinvestigation?
A.Bonescan B.CTscanoftheabdomen C.MRIscanofthespine D.CTscanofthebrain E.MRIofthebrain

Answer:c)MRIscanofthespine. Thispatientislikelytohavecordcompressionfromametastasestothespine. UrgentMRIscan isrequiredtoconfirm the diagnosis and a referralforeither surgery or radiotherapycanthenbemade.
Dr.KhalidYusufElzohrySohagTeachingHospital2012 117

www.MRCPass.com

Neurology

NeurologyQ117
A 30 year old man had a progressive four month history of stiffness, tremors and unsteadiness.Thereisnofamilyhistoryofasimilarproblem. On examination, there is globally increased tone. Power is normal and there are brisk reflexes throughout. A resting tremor and some abnormal writhing movements in the armswereobserved.HehasanMMSEscoreof25/30. Whatisthemostlikelydiagnosis?
A.Motorneurondisease B.Polymyalgiarheumatica C.Friedrich'sataxia D.Wilson'sdisease E.Temporalarteritis

Answer:d)Wilson'sdisease. Neuroacanthocytosis,Wilson's,Huntington's,paraneoplasticsyndromecanpresentwith choreiform movements and cognitive impairment. Cerebellar signs are more typical in Friedrich'sataxia.
Dr.KhalidYusufElzohrySohagTeachingHospital2012 118

www.MRCPass.com

Neurology

NeurologyQ118
A60yearoldmandevelopsswallowingdifficultiesandcomplainsofdifficultywalkingup stairs. On examination,there is weakness and wastingofthedistal muscles of the arms. There are visible fasciculations. Reflexes are brisk and the plantars are upgoing. There are no sensory abnormalities. Speech and language assessment suggests the presence of pharyngealmuscleweakness. Whatisthemostlikelydiagnosis?
A.GuillainBarre B.Motorneurondisease C.Myastheniagravis D.Chronicinflammatorydemyelinatingpolyneuropathy E.Paraneoplasticsyndrome

Answer:b)motorneurondisease. Thisisaclassicalpresentationofmotorneurondisease.Thereisapseudobulbarpalsyin association with fasciculations of the muscles and a mixture of upper and lower motor neuronsigns.
Dr.KhalidYusufElzohrySohagTeachingHospital2012 119

www.MRCPass.com

Neurology

NeurologyQ119
A 35yearold female was admitted to the hospital after experiencing a sudden and severeepisodeofheadache. No precipitating factor was identified. Except for mild neck stiffness, general physical andneurologicalexaminationswereunremarkable. Onadmission,computedtomography(CT)scanshowedathincollectionofbloodonthe rightsylviancistern,establishingthediagnosisofsubarachnoidhaemorrhage. Theadmittingdoctorisconsideringfurthermanagement. Which of the Following medications have been shown to decrease the incidence of cerebralinfarctioninpatientswithsubarachnoidhaemorrhage?
A.Amlodipine B.Prednisolone C.Nimodipine D.Acetazolamide E.Bendrofluazide

Answer:c)nimodipine. Nimodipine (calcium channel blocker) has been shown to improve outcome in patients withsubarachnoidhaemorrhage.Itisgiven60mgorally,4hourly.
Dr.KhalidYusufElzohrySohagTeachingHospital2012 120

www.MRCPass.com

Neurology

NeurologyQ120
A 28 year old woman is assessed for easy fatigue. She complains of weakness of her handscausingpoorcoordinationanddoublevision.Thisisworstintheevenings. Whatisthediagnosis?
A.Guillainbarresyndrome B.Multiplesclerosis C.Myastheniagravis D.Paraneoplasticsyndrome E.Centralpontinemyelinolysis

Answer:c)Myastheniagravis. Myasthenia gravis is an acquired autoimmune disorder characterised by weakness, typicallyoftheperiocular,facial,bulbar,andgirdlemuscles.Diplopia,ptosisandslurring of the speech are common symptoms. There is typically fatiguabulity on examination, andsymptomsareworseduringtheevenings.ItisassociatedwithserumIgGantibodies to acetylcholine receptors in the postsynaptic membrane of the neuromuscular junction.
Dr.KhalidYusufElzohrySohagTeachingHospital2012 121

www.MRCPass.com

Neurology

NeurologyQ121
A 35 year old lady has developed progressive weakness in the hands and feet over a week. There is some associated numbness and tingling in the hands and feet. She complainsofbreathingdifficulty.Shehadseveralepisodesofdiarrhoeatwoweeksago. On examination, there is slurring of speech. Her reflexes are all absent and there is sensorylossinagloveandstockingpattern. Whichisthenextappropriatestep?
A.MRI B.Lumbarpuncture C.Vitalcapacity D.EMG E.AntiAchantibody

Answer:c)vitalcapacity. Guillain Barre syndrome is the likely diagnosis. As she is complaining of breathlessness, vital capacity measurement would be essential because of potential rapid deterioration inrespiratoryfunction.
Dr.KhalidYusufElzohrySohagTeachingHospital2012 122

www.MRCPass.com

Neurology

NeurologyQ122
A40yearoldpatienthasbeendiagnosedwithmigrainerecently.Themigrainesseemto haveincreasedinfrequencysincebeingonatriptan. Whatisthenextmedicationtouse?
A.Propanolol B.Tramadol C.Carbamazepine D.Neurofen E.Aspirin

Answer:a)propanolol. In migraines, beta blockers, calcium channel blockers and antidepressants may be helpfulinprophylaxis.
Dr.KhalidYusufElzohrySohagTeachingHospital2012 123

www w.MRCPass.c com

Ne eurology

Ne eurologyQ123
A 50 5 year old d man pre esents with h speech difficulty. d He H has dif fficulty with word iden ntificationand a repetition.Heisab bletopickout o correctobjectswhe eninstructe edtodo soand a isnotco onfused. ereisthele esion? Whe
A.Super riorfrontallo obe B.Pariet tallobe C.Media alsuperiorte emporallobe e lobe(Wernic D.Poste erior,superio ortemporall ckesarea) E.Inferio orfrontallob be(Brocasa area)

Answer:e)Infe eriorfrontal llobe(Broc casarea). Brocas Area (Inferior frontal lo obe) dama age causes impaired d fluency, intact com mprehension n,impairedrepetition [expressive edysphasia] ]. Wernickes Ar rea (posteri ior, superio or tempora al lobe) dam mage cause es normal fluency, f imp pairedcomp prehension,impairedrepetition[r receptivedy ysphasia].

Ne eurologyQ124
Dr.Khalid K Yusuf fElzohrySo ohagTeachin ngHospital 2012 124

www.MRCPass.com

Neurology

A 60 year old man has presented with an episode of transient visual loss lasting for 5 minutes.Hemaintainedconsciousnessthroughout.ACTscandidnotshowanychanges. Hementionsthatheisdrivingcurrently. How long is a patient required to avoid driving a car Following recovery from a single transientischaemicattack?
A.6weeks B.1month C.3months D.6months E.1year

Answer:b)1month. Following a TIA, a patient should not drive for 1 month. If there were recurrent TIAs, thenthepatientshouldbesymptomfreefor3monthsbeforedriving. Followingaseizurewithalteredconsciouslevel,apatientshouldnotdrivefor1year.All casesshouldbereportedtotheDriverVehicleLicensingAgency(DVLA).
Dr.KhalidYusufElzohrySohagTeachingHospital2012 125

www.MRCPass.com

Neurology

NeurologyQ125
A 40 year old man complained of severe headache. Following this, he then developed nauseaand vomiting and cametoA+E to be assessed.A CT scandemonstratedblood in theleftsylvianfissure. WhichoftheFollowingisthemostlikelycause?
A.Migraine B.Subduralhaematoma C.Sagittalsinusthrombosis D.Carotiddissection E.Berryaneurysmrupture

Answer:e)berryaneurysmrupture. The history is consistent with a subarachnoid haemorrhage, caused by rupture of a berry(saccular)aneurysm.
Dr.KhalidYusufElzohrySohagTeachingHospital2012 126

www w.MRCPass.c com

Ne eurology

Ne eurologyQ126
A 46 year old man had su uffered from m classical seropositiv ve rheumatoid arthritis s for 10 years. He has never rece eived treatm ment with phenylbuta azone, gold d, or antimalarials. inte ense pain in n the right hand. Over r a course of o 3 weeks, he noticed pain, num mbness, and tingling on n the latera al aspect of the palm and a in the fourth f and fifth fingers of the ht hand. On n examinati ion, he has s weakness in the abd ductors and d adductors s of the righ fing gers,andhis shandishe eldinaclaw wedposition n. atisthedia agnosis? Wha
A.Comm monperonea alnervelesio on B.Latera alcutaneous snervelesion C.Radialnervelesio on D.Mediannervelesion E.Ulnarnervelesion n

Answer:e)ulna arnerveles sion. The e ulnar nerv ve supplies s the lumbr ricals (claw w hand) and d also the Interossei Dorsal: Abd ductors(DAB)andPalm mar:Adduct tors(PAD). A le esion in th he anterior interosse eous nerve results in n weakness s of the terminal t phalangesofth hethumbandindexfin ngers.

Claw wHand

Dr.Khalid K Yusuf fElzohrySo ohagTeachin ngHospital 2012 127

www.MRCPass.com

Neurology

NeurologyQ127
A65yearoldmanhasrecentlybeeninvolvedinanaccident.Onexamination,therewas increased tone in the right leg, he had right sided leg weakness and loss of proprioception on the right side of the lower limb. There was also left sided loss of sensationtopainontheleftleg. Whatisthediagnosis?
A.Myalgicencephalitis B.Motorneurondisease C.BrownSequardsyndrome D.CreutzfeldtJakobdisease E.Subacutecombineddegenerationsyndrome

Answer:c)BrownSequardsyndrome. BrownSequardorhemisectionofthespinalcordcausesinlossofsensationtopainand temperature contralateral to the lesion, with ipsilateral loss of sensation for position andvibration,anduppermotorneuronparalysisipsilateraltothelesion.
Dr.KhalidYusufElzohrySohagTeachingHospital2012 128

www.MRCPass.com

Neurology

NeurologyQ128
A 38 year old lady has difficulty walking. On examination, she has an ataxic gait, increasedtoneinthelegswithhyperreflexiaandmotorweakness. Whatdiagnosticinvestigationshouldbearranged?
A.CTscanofthebrainandspine B.MRIofbrainandspine C.LumbarspineXray D.EMG E.EEG

Answer:B)MRIofbrainandspine. The diagnosis is likely to be multiple sclerosis. MRI would be useful to assess for demyelinating lesions. Following this lumbar puncture is also helpful to confirm the presenceofoligoclonalbands(whencomparedtoserum).
Dr.KhalidYusufElzohrySohagTeachingHospital2012 129

www w.MRCPass.c com

Ne eurology

Ne eurologyQ129
A18 8yearoldman m hasdiff ficultyinwalking.Hehas h hadpro oblemswith hvisionwhic chwere diffi iculttocorr rectwithgla asses. On examinatio on, there is nystagmus s and pallor of the op ptic discs. He H has a dy ysarthria and intention tremor. There are abs sent reflexe es in the kn nees and ankles. Plant tars are equ uivocal.Pescavuswasn noted. atisthemo ostlikelydia agnosis? Wha
A.Spino ocerebellarataxia B.Friedr rich'sataxia C.Alcoh holiccerebell lardegenera ation D.Wilso on'sdisease E.Hered ditarysensor rimotorneur ropathy

Answer:b)Frie edrich'satax xia. Frie edrich's At taxia is a an autosom mal recess sively inhe erited. Pat tients may y have card diomyopath hy and diab betes. Neurological abnormalitie es include optic atrop phy and retinitis pigme entosa, nystagmus, ce erebellar disease and signs, loss of dorsal column sens sationandweakness. w P Pescavus(h higharched dfeet)isusu uallypresen nt.

PesCavus

Dr.Khalid K Yusuf fElzohrySo ohagTeachin ngHospital 2012 130

www.MRCPass.com

Neurology

NeurologyQ130
A65yearoldmanhasbeenadmittedFollowinganepisodeofconfusion.Hewasontwo different diuretic drugs on admission which were discontinued due to hyponatraemia. On day 1, he had a sodium of 112 mmol/l and on day 3 he had a sodium of 145 mmol/. Hehaddevelopedbyday3,spasticquadriparesisandpseudobulbarpalsy. Whatisthelikelydiagnosis?
A.Multiplesclerosis B.Tuberculousmeningitis C.Motorneurondisease D.Centralpontinemyelinolysis E.Glioma

Answer:D)centralpontinemyelinolysis. The upper motor neuron features and rapid correction of hyponatraemia suggests central pontine myelinolysis. The neurological features may improve gradually but certain patients have permanent neurological deficits from rapid changes in serum sodium.
Dr.KhalidYusufElzohrySohagTeachingHospital2012 131

www w.MRCPass.c com

Ne eurology

Ne eurologyQ131
A 40 0 year old man m compla ains of pain n in the righ ht eye, whic ch had wors sened over the last few wdays. On examinatio on, he had a ptosis of r right eye. He H also had weakness of superior upgaze oftherighteye eandlosso ofaccommo odationrefle ex. Whi ichoftheFo ollowingislikely? l
A.Myas stheniagravis B.Neuro osyphilis C.Multiplesclerosis D.Poste eriorcommunicatingarte eryaneurysm m E.Myoto onicdystrop phy

Answer:d)pos steriorcomm municatingarteryaneu urysm. Pati ients with painful, p isola ated thirdnerve n palsy withpupillary involvement is mo ost likely due e to a post terior comm municating g artery an neurysm. Other causes of mono oneuritis mul ltiplex (diab betes, vascu ulitis, syphilis), can als so cause a third t nerve palsy, but seldom arepainful.

Arte eriogramLateral L view wshowingaposteriorcommunica c tingarteryaneurysm.

Dr.Khalid K Yusuf fElzohrySo ohagTeachin ngHospital 2012 132

www.MRCPass.com

Neurology

NeurologyQ132
A 55 year old man who was a heavy smoker in the past complains of arm weakness. On examination, there was postural hypotension. He has proximal muscle weakness with fatiguabilityandlossoftendonreflexes. Whatisthelikelydiagnosis?
A.Motorneurondisease B.Myastheniagravis C.Transversemyelitis D.GuillainBarresyndrome E.LambertEatonmyasthenicsyndrome

Answer:e)LambertEatonmyasthenicsyndrome. In Lambert Eaton myasthenic syndrome, 60% of cases are paraneoplastic (small cell lung ca is most associated). The clinical features are proximal weakness, loss of tendon reflexesandautonomicdysfunction.
Dr.KhalidYusufElzohrySohagTeachingHospital2012 133

www w.MRCPass.c com

Ne eurology

Ne eurologyQ133
A 60 0 year old man m has a r right wrist d drop. On ex xamination he has an absent trice eps jerk on the t right as s well as an area of sensory loss over o the do orsum of the middle finger on therighthand. . Whe ereisthele esion?
A.Mediannerve B.Radia alnerve C.Brach hialnerve D.Ulnar rnerve E.Musculocutaneou usnerve

Answer:b)radialnerve. Rad dial nerve dysfunction d involves im mpaired mo ovement or r sensation of the back k of the arm m (triceps), the t forearm m, or the ha and caused by damage e to the rad dial nerve. There T is wea aknessofwristandelb bowextension,andabs senttricepsjerk.

WristDrop

Dr.Khalid K Yusuf fElzohrySo ohagTeachin ngHospital 2012 134

www w.MRCPass.c com

Ne eurology

Ne eurologyQ134
A 24 2 year old d tennis pl layer find that he is unable to o extend his h right wr rist. On examination,thereiswea aknessoftheextensors softhewris standdigits s. Whi ichoftheFo ollowingstr ructuresisd damaged?
A.Radia alnerve B.Musc culocutaneou usnerve C.Media annerve D.Medialcordofthebrachialpl lexus E.Latera alcordofthe ebrachialple exus

Answer:a)radi ialnerve. The eradialnerv vemaybedamaged d an nywherein nitscourse.Itismostcommonly c affected a in the t upper arm where e it winds round the humerus and in the e extensor muscle com mpartment of the forearm affecting the posterior inter rosseous br ranch. Radia al nerve pals sy causes weakness w of o the wrist t and elbow w extension, and wea akness of forearm f supination.Sen nsorylosso onthedorsu umofhandandforearm mmayalsobepresent t.

Rad dialNervePalsy

Dr.Khalid K Yusuf fElzohrySo ohagTeachin ngHospital 2012 135

www.MRCPass.com

Neurology

NeurologyQ135
A 50 year old lady has a painful cheek (maxillary area) when chewing. The neurologist diagnosestrigeminalneuralgia. Whichisthebestmedicationtotry?
A.Benztropine B.Tramadol C.Diclofenac D.Propanolol E.Carbamazepine

Answer:e)carbamazepine. For trigeminal neuralgia, carbamazepine is the first treatment of choice, but baclofen canalsobetried.
Dr.KhalidYusufElzohrySohagTeachingHospital2012 136

www w.MRCPass.c com

Ne eurology

Ne eurologyQ136
A 46 6 year old woman w com mplains of diplopia. d On n examinat tion, she ha as double vi ision on look kingtotheleftonly. The ere is failur re of adduc ction in the e right eye e, and nysta agmus in the t left eye e whilst look kingtotheleft. ichoneofth heFollowin ngislikely? Whi
A.Super riorrectuspalsy B.Inferiorrectuspalsy C.Latera alrectuspals sy D.Mediallongitudin nalfasciculus sdefect E.Inferio orcerebellar rpedunclele esion

Answer:D)mediallongitudinalfascic culusdefect t. Dipl lopia on ey ye abduction is likely t to be due to t either a lateral rect tus palsy or r medial long gitudinal fasciculus f (MLF) defect. This scenario fits one of inter rnuclear opthalmoplegi ia (which is s due to an MLF defect). In this scenario it is s a right sid ded INO sam mesideasth heeyewhic chfailstoad dduct.

Inte ernuclearop pthalmoplegia


Dr.Khalid K Yusuf fElzohrySo ohagTeachin ngHospital 2012 137

www w.MRCPass.c com

Ne eurology

Ne eurologyQ137
A 35 year old patient wit th HIV is as ssessed for new onset right arm weakness. An MRI scan nofhisheadshowsatemporo t pa arietalringenhancing e lesion. Whi ichisthemostlikelyca auseforthis s?
A.Lymp phoma B.Toxop plasmosis C.Behce et'sdisease D.CMV E.HSV

Answer:b)toxo oplasmosis. Cere ebral toxop plasmosis is s the most common cause c of a ring r enhanc cing lesion causing neu urologicalde efectinaHIVpatient.

Cere ebralToxop plasmosis


Dr.Khalid K Yusuf fElzohrySo ohagTeachin ngHospital 2012 138

www.MRCPass.com

Neurology

NeurologyQ138
A45yearoldladyiswithasuddenonsetofsevereheadache.Itcameonwitha'bang'.A CT scan of the brain is reported as normal. There are no neurological signs on examination. Thenextbestmanagementstepisto:
A.Startergotamine B.MRIofthebrain C.Performalumbarpunctureimmediately D.Performalumbarpuncture24hoursaftertheonsetofheadache E.RepeatCTscan24hourslater

Answer:d)performalumbarpuncture24hoursaftertheonsetofheadache. Lumbar puncture should be performed in suspected SAH (ideally after 24 hours from onset of headache) if the CT scan is not diagnostic. The CSF specimen should be examined by spectrophotometry for the presence of xanthochromia due to the presenceofoxyhaemoglobinandbilirubin.
Dr.KhalidYusufElzohrySohagTeachingHospital2012 139

www.MRCPass.com

Neurology

NeurologyQ139
A40yearoldmanwasinjuredinaroadtrafficaccident.Onexamination,heisunableto extend his right hand at the wrist. The triceps jerk is diminished, and there is weakness of wristflexion.A smallareaofover therightmiddlefingerhas sensorylosstopainand touch. Whereisthelesion?
A.Radialnerve B.Ulnarnerve C.C5root D.C7root E.T1root

Answer:d)C7root. A C5 root lesion causes weakness in abduction of the shoulder and biceps, as well as sensorylossin the upperarm.A C7root lesion causes weakwristextensors andflexors, weakfingerextensorsandsensorylosstomiddlefinger.Aradialnervelesionwouldnot involvefingerflexors.AT1lesionwouldcauseweakintrinsichandmuscles.
Dr.KhalidYusufElzohrySohagTeachingHospital2012 140

www.MRCPass.com

Neurology

NeurologyQ140
A 62 year old woman presents with diplopia. On examination, she has a fixed dilated pupilontherightside.Directlightreflexandaccommodationreflexaretotallyabsent. WhichoneoftheFollowingismostlikely?
A.ArgyllRobertsonpupil B.Hornerssyndrome C.3rdnervepalsy D.Pilocarpine E.Uveitis

Answer:C)3rdnervepalsy. Pilocarpineis a miotic, and allthe rest are causes of small pupils as well. Although there is no mention of ptosis or oculomotor nerve palsy, this is the only answ er which fits withafixeddilatedpupil
Dr.KhalidYusufElzohrySohagTeachingHospital2012 141

www w.MRCPass.c com

Ne eurology

Ne eurologyQ141
A 6 60 year old man has previously diagnosed with pernicious anae emia. He was w non com mpliant with h medicatio on for seve eral years. He now ha as unsteadiness and difficulty d walking. Whi ichoftheFo ollowingles sionsislikel ly?
A.Corpu uscallosum B.Spino othalamictra act C.Dorsa alcolumns D.Thala amus E.Basalganglia

Answer:c)dors salcolumns s. The e diagnosis is Subacute e Combined d Degenera ation of the e cord. The posterior columns c and corticospin nal tracts are specifica ally damage ed, but the clinical pict ture is complicated byt theearlyde evelopmentofcoexiste entperipher ralnerveda amage.

Dr.Khalid K Yusuf fElzohrySo ohagTeachin ngHospital 2012 142

www.MRCPass.com

Neurology

NeurologyQ142
A25yearoldladypresentswithtwomonth'shistoryofepisodicvisuallossaffectingthe right eye. Over the last two years, she had gained a considerable amount of weight. Examination showed bilateral optic disc swelling and small retinal haemorrhages in the righteye. Whichdiagnosisislikely?
A.Multiplesclerosis B.Grave'sopthalmopathy C.Retinitispigmentosa D.Benignintracranialhypertension E.KearnSayre'sdisease

Answer:d)benignintracranialhypertension. A high BMI is associated with BIH. The condition can be worsened by tetracycline and oralcontraceptivepills.
Dr.KhalidYusufElzohrySohagTeachingHospital2012 143

www.MRCPass.com

Neurology

NeurologyQ143
A 60 year old lady presents with back pain that radiates to the knee and down the medialsideofthecalftowardsthemedialmalleolus. Whichnerverootisaffected?
A.L2 B.L3 C.L4 D.L5 E.S1

Answer:c)L4. In contrast, L5 lesion can cause pain radiating through the buttock, down the postero lateralaspectofthethigh,lateralaspectofcalfandacrossthedorsumofthefoot.
Dr.KhalidYusufElzohrySohagTeachingHospital2012 144

www w.MRCPass.c com

Ne eurology

Ne eurologyQ144
A21 1 yearoldman m presen nts with afe everandhe eadache.His stemperatu urepersists s andhe now w complains s of a bad headache. h H He is disorie entated in c casualty where he is as ssessed. Dur ring examin nation, he has a gene eralized seizure. A CT scan is pe erformed and it is normal. CSF examination shows a p protein of 0.3g and 10 1 white ce ells, predom minantly A EEGshow wedperiodicsharpwaveactivity. lymphocytes.An atisthemo ostlikelydia agnosis? Wha
A.Meningococcalmeningitis m B.TBme eningitis C.HSVencephalitis e D.Progr ressivemultifocalleucoe encephalopat thy E.Listeriameningitis s

Answer:c)HSV Vencephalit tis. HSV V encephali itis is the most likely y there can c be diso orientation or dysphasia and seiz zures. There e is also mild lymphocytosis on the CSF. In n HSV ence ephalitis, MRI M may show frontal or temporal lobe involv vement. EEG G may show w periodic sharp s wave activity tem mporallyand dbackgroun ndoffocalo ordiffuseslowing.

Righ httemporal llobeinvolv vementinHSV H enceph halitis


Dr.Khalid K Yusuf fElzohrySo ohagTeachin ngHospital 2012 145

www.MRCPass.com

Neurology

NeurologyQ145
A 65 year old man is assessed on the ward for weakness in his legs. He is an ex smoker and drinks 15 units of alcohol in a week. His wife mentions that he is confused. On examination,hisMMSEscoreis20/30.Hehasanataxicgait.Thereisbilateralpyramidal weakness and coordination is impaired. Routine blood tests are normal. An MRI scan of the head shows diffuse white matter changes, more in the cerebellar region than the cerebrum. Whichofthesetestswouldhelpmostinconfirmingthediagnosis?
A.CSFforoligoclonalbands B.CSFforantiHuandantiYoantibodies C.CSFforTBculture D.EEG E.EMG

Answer:b)CSFforAntiHuandantiYoantibodies. Anti Hu and anti Yo antibodies would help confirm a diagnosis of paraneoplastic syndrome.Multiplesclerosisisunlikelyinviewoflatepresentationandisnotcommonly associatedwithdementia.
Dr.KhalidYusufElzohrySohagTeachingHospital2012 146

www w.MRCPass.c com

Ne eurology

Ne eurologyQ146
A 45 5 year old lady has numbness in the right lit ttle finger and a aspect of o the palm m shown inth hepicture.The T smallmuscles m ofthehandare eweak.

ichnerveisaffected? Whi
A.Mediannerve B.Radia alnerve C.Ulnar rnerve D.Anter riorinterosse eousnerve E.Posteriorinteross seousnerve

Answer:c)ulna arnerve. The e ulnar nerv ve innervate es the third and fourth h lumbricals s, the intero ossei and adductor pollicis.Sensationissuppl liedtothef fifthfingerand a theulna arpartofth hefourthfinger.
Dr.Khalid K Yusuf fElzohrySo ohagTeachin ngHospital 2012 147

www.MRCPass.com

Neurology

NeurologyQ147
A 35 year old man has bradykinesia and rigidity of his limb movements. He was also noticed to have choreiform movements when observed. He has a history of poor developmentduringthelatterpartofschooleducation. Whatisthemostlikelydiagnosis?
A.NewvariantCJD B.Parkinsonsdisease C.Alzhheimersdisease D.Normalpressurehydrocephalus E.Huntingtonsdisease

Answer:E)Huntingtonsdisease. In Huntingtons disease, as the disease progresses, chorea coexists with andgradually is replaced by dystonia and parkinsonian features, such as bradykinesia, rigidity, and postural instability. It is also associated with cognitive impairment as well as psychiatric manifestations.
Dr.KhalidYusufElzohrySohagTeachingHospital2012 148

www.MRCPass.com

Neurology

NeurologyQ148
A 65 year old man has had a 6 month history of progressiveworsening of confusion. He hadleftthecookeronseveraltimesandaccusedtheneighboursofspyingonhim. On examination, his face is expressionless and he has a monotonous speech. Cranial nervearenormal. Increasedtoneispresentinall4limbs.Reflexes,powerandsensationareallnormal. Whatisthemostlikelydiagnosis?
A.Parkinson'sdisease B.Lewybodydementia C.Alzheimer'sdementia D.Huntington'schorea E.CreutzfeldtJakobdisease

Answer:b)Lewybodydementia. This patient has parkinsonism, with bradykinesia and rigidity. In view of the deterioration in mental function and delusion,Lew y body dementia is more likely than Parkinson'sdisease.
Dr.KhalidYusufElzohrySohagTeachingHospital2012 149

www.MRCPass.com

Neurology

NeurologyQ149
A 74 year old lady lives alone and is self caring. She has home help twice a week. two days ago, she was found wandering the street and appears confused. The ambulance crewwhobroughthertohospitalhasnoticedthatshehasbeenincontinentofurine. On examination, she walks with a wide based gait and has an MMSE score of 17 / 30. Hertemperatureis36.9CandshehasaCRPof30mg/l. Whatisthemostlikelydiagnosis?
A.Urinarytractinfection B.Subduralhaematoma C.Drugoverdose D.Normalpressurehydrocephalus E.Alzheimersdisease

Answer:D)normalpressurehydrocephalus. Normal pressure hydrocephalus (NPH) is a clinical symptom complex characterized by abnormal gait, urinary incontinence, and dementia. The CRP is not specific and there is insufficientevidenceforaUTIinthiscase.
Dr.KhalidYusufElzohrySohagTeachingHospital2012 150

www.MRCPass.com

Neurology

NeurologyQ150
A 50 year old man presents with weakness in his arms and legs which is worse at the endoftheday.Onexaminationhehasbilateralptosisanddecreasedreflexes. WhattestwillconfirmthathismuscleweaknessisduetoMyastheniaGravisratherthan LambertEatonmyasthenicsyndrome?
A.Tensilontest B.Trialofpyridostigmine C.Trialofprednisolone D.EMG E.Musclebiopsy

Answer:d)EMG. The main differential is myasthenia gravis and LEMS. Myasthenia gravis can be differentiated from Eaton Lambert myasthenic syndrome by electromyography. Repetitive stimulation in myasthenia gravis leads to a decrement of evoked muscle action potentials, whilst in myasthenic syndrome the condition improves by repetitive stimulation.
Dr.KhalidYusufElzohrySohagTeachingHospital2012 151

www.MRCPass.com

Neurology

NeurologyQ151
A 60 year old patient has been confused for 1 week. His wife also mentions he had two episodes of tonic clonic seizures. He has a recent diagnosis of smallcell lung cancer whichistreatedwithradiotherapy. Whatisthediagnosis?
A.Myastheniagravis B.Meningitis C.Paraneoplasticsyndrome D.Cerebellarstroke E.Wilson'sdisease

Answer:c)paraneoplasticsyndrome. A wide variety of paraneoplastic neurological manifestations have been described, but parkinsonism is uncommon. Cerebellar degeneration, tremor and movement disorders, andLambertEatonMyasthenicsyndromesarethecommonest.
Dr.KhalidYusufElzohrySohagTeachingHospital2012 152

www.MRCPass.com

Neurology

NeurologyQ152
A patient has marked dizziness and unsteadiness during walking. On examination, he hasaleftsidedHorner'ssyndromeandleftsidedweakness.Thereislossofsensationto pinprickontherightside. Whatisthelikelydiagnosis?
A.Leftinternalcapsuleinfarct B.Posteriorinferiorcerebellararteryocclusion C.Medullaryinfarct D.Multiplesclerosis E.Vertebralarterydissection

Answer:b)posteriorinferiorcerebellararteryocclusion. AlsoknownasWallenberg'ssyndrome,thesignsare vertigo, ipsilateral cerebellar signs and weakness, and contralateral sensory loss. There isalsocranialnerve involvementcausingdysphagiaanddysarthria.
Dr.KhalidYusufElzohrySohagTeachingHospital2012 153

www.MRCPass.com

Neurology

NeurologyQ153
A 55 year old man has type 2 diabetes. He complains of difficulty walking. On examination, he is unable to evert his right foot. He has intact knee reflexes. There is alsosensorylossinthelateralaspectofthefoottopinprick. Whichnerverootisinvolved?
A.L2 B.L3 C.L5 D.S1 E.S2

Answer:C)L5. Thecommonperonealnerveisinvolved.NerverootsL5andS1supplythenerveforfoot eversion. Thecutaenoussensorysupplycorrespondstothelateralpartofthefoot.


Dr.KhalidYusufElzohrySohagTeachingHospital2012 154

You might also like